Download as pdf or txt
Download as pdf or txt
You are on page 1of 123

 

 
Copyrighhts © 2016 by
b the Deparrtment of Clin
nical Pharma
acology at Fa
aculty of Meedicine, Mnas
soura
Universitty, Egypt.

Previouss editions copyright © 20


015, 2014, 20
013, 2012, 2011, 2010, 2009,
2 2008, 22000 by the
Departm
ment of Cliniccal Pharmacoology at Facu cine, Mnasou
ulty of Medic ura Universitty, Egypt.

No part of this book may be reprroduced or d distributed in


n any form orr by any meaans, or stored
d in a
databasee or retrievall system, witthout the prio
or written pe
ermission of the
t copyrighhts owner,
Departm
ment of Cliniccal Pharmaco ology at Facuulty of Mediccine, Mansouura Universitty.

This is a copyrightedd work and is s protected b


by the Egypttian Intellectu
ual Property Law 82 of 2002. Use
of this w
work is subjec
ct to this law w. The Departrtment of Clin
nical Pharmaacology at MMansoura Fac culty of
Medicine e reserves alll rights in an
nd to the worrk.

2000 ‫سنة‬
‫ لس‬1456 :‫رقم اإليداع بدار الككتب‬
‫م‬
2000/9/6 ‫ بتاريخ‬ 
Preface

C
linical training for undergraduate students often focuses on diagnostic rather than
therapeutic skills. Sometimes students are only expected to copy the prescribing
behavior of their clinical teachers, or existing standard treatment guidelines, without
explanation as to why certain treatment is chosen. Books may not be much help either.
Pharmacology reference works and formularies are drug-centered, and although clinical
textbooks and treatment guidelines are disease-centered and provide treatment
recommendations, they rarely discuss why these therapies are chosen. Different sources
may give contradictory advice.
This book in primarily intended for under graduate medical students who are about to
enter the clinical phase of their studies. It will provide step by step guidance to the process
of rational prescribing together with many illustrative examples. It teaches also skills that are
necessary throughout a clinical career. Postgraduate students and practicing doctors may
also find it a source of straightforward information.
I wish to acknowledge the ongoing efforts of my contributing authors, and we are deeply
grateful to all those who have with such good grace given us their time and energy to supply
valuable facts and opinions, they principally include:

 Prof. Hussein El-Beltagi who took over the preparation of all books since the 1st edition
in 1995 including the revision process, printing control, distribution and selling control.
 Assist. Prof. Mohamed-Hesham Daba who took over the revision process and
amendments of the last two editions.
 Assist. Prof. Abdel-Motaal Fouda who prepared the last two editions in a readable up-
to-date text to provide essential information necessary throughout the clinical career.
 Dr. Sameh Abdel-Ghany who assisted in the revision process.

Much of any merit this book may have is due to the


generosity of those named above.

Gamal M. Dahab (MD, PhD)

Professor Emeritus in Clinical Pharmacology


Mansoura Faculty of Medicine
 

iii
Miss
sion and
a Vis
sion

Our mission
The Clinnical Pharm macology Department
D is seeking excellence
e and leadeership in fou
ur major
core acctivities: edu
ucation, ressearch, com
mmunity serrvice, and faculty
f and staff development.
We are e connectin ng basic medical
m sc iences with clinical care
c througgh innovattive and
disciplin
ned teachin ng of clinica
al pharmaco
ology in an integrative
i manner
m

Our v
vision
The dep partment off Clinical Ph
harmacolog gy is aiming
g to be a premier acad demic model in the
field of pharmacoloogy and the erapeutics in Egypt annd Middle East
E throug h promoting use of
the bestt therapeutics and dev veloping new
wer experimmental and clinical reseearch proje
ects.

Value
es

The gu
uiding prin
nciples and beliefs ffor the dep
partment

 Excellence, creativity, innovation, fairness, honesty, transparenncy, collab


boration,
teammwork and lifelong learning
 Reccognition tha
at our stude
ent comes ffirst
 All m
members ofo our department musst see them mselves as integral to the successs of our
misssion and ouur departmeent as integ ral to their personal
p su
uccess.
 As we subscrribe to the ese values,, we shall be profes ssionals in the profes
ssion of
education.

iv
Contributers

Effat A. Haroun MD, PhD Somaya A. Mokbel MD, PhD


Prof. of Clin Pharmacology Prof. of Clin Pharmacology
Mansoura Faculty of Medicine Mansoura Faculty of Medicine

Elhamy M. El-Kholy MD, PhD Amany A. Shalaby MD, PhD


Prof. of Clin Pharmacology Prof. of Clin Pharmacology
Mansoura Faculty of Medicine Mansoura Faculty of Medicine

Gamal M. Dahab MD, PhD, MSc (Int.Med) Amal Abdel-Hamid MD, PhD
Prof. of Clin Pharmacology Prof. of Clin Pharmacology
Mansoura Faculty of Medicine Mansoura Faculty of Medicine

Farida M. El-Banna MD, PhD Essam A. Ghyati MD, PhD


Prof. of Clin Pharmacology Assist. Prof. of Clin Pharmacology
Mansoura Faculty of Medicine Mansoura Faculty of Medicine

Aly M. Gaballah MD, PhD, MSc (int.Med) Mohamed-Hesham Y. Daba MD, PhD
Prof. of Clin Pharmacology Assist. Prof. of Clin Pharmacology
Mansoura Faculty of Medicine Mansoura Faculty of Medicine

Layla T. Hanna MD, PhD Abdel-Motaal M. Fouda MD, PhD


Prof. of Clin Pharmacology Assist. Prof. of Clin Pharmacology
Mansoura Faculty of Medicine Mansoura Faculty of Medicine

Mohamed Kheriza MD, PhD, MSc (Int.Med) Vivian Boshra MD, PhD
Prof. of Clin Pharmacology Assist. Prof. of Clin Pharmacology
Mansoura Faculty of Medicine Mansoura Faculty of Medicine

Abdel-Rahman A. Yassin MD, PhD Hala A. Al-Ashri MD, PhD


Prof. of Clin Pharmacology Assist. Prof. of Clin Pharmacology
Mansoura Faculty of Medicine Mansoura Faculty of Medicine

Mohmmad A. Attia MD, PhD Nageh Rizk MD, PhD


Prof. of Clin Pharmacology Lecturer in pharmacology
Mansoura Faculty of Medicine Mansoura Faculty of Medicine

Mohamed Abdel-Ghani MD, PhD Elsayed A. Hassan MD, PhD


Prof. of Clin Pharmacology Lecturer in Clin Pharmacology
Mansoura Faculty of Medicine Mansoura Faculty of Medicine

Hussien M. El-Beltagi MD, PhD Mohamed Abdel-Monem MD, PhD


Prof. of Clin Pharmacology Lecturer in Clin Pharmacology
Mansoura Faculty of Medicine Mansoura Faculty of Medicine

Karawan M. Abdel-Rahman MD, PhD Mahmoud A. Naga MD, PhD


Prof. of Clin Pharmacology Lecturer in Clin Pharmacology
Mansoura Faculty of Medicine Mansoura Faculty of Medicine

  v
 
Ahmad Hassan MD, PhD Mohamed Abou El-khair MD, PhD
Lecturer in Clin Pharmacology Lecturer in Clin Pharmacology
Mansoura Faculty of Medicine Mansoura Faculty of Medicine

Ahlam El-masry MD, PhD Sameh A. Abdel-Ghani MSc.


Lecturer in Clin Pharmacology Assist. Lecturer in Clin Pharmacology
Mansoura Faculty of Medicine Mansoura Faculty of Medicine

Rehab Hamdy MD, PhD


Lecturer in Clin Pharmacology
Mansoura Faculty of Medicine

vi  
 
Table of Contents
 

CHAPTER 1: GENERAL PRINCIPLES

Part 1: Pharmacodynamics 1
Receptors 2
Ion channels 7
Enzymes 8
Carrier molecules 8
Part 2: Factors affecting the dose-response relationship 8
Factors related to the drug 8
Factors related to the patient 10
Part 3: Clinical pharmacokinetics 13
Absorption of drugs 13
Distribution of drugs 15
Elimination of drugs 16
Metabolism of drugs 20
Part 4: Adverse drug reactions 23
Drug induced liver injury 24
ADR on pregnancy 25
Part 5: Principles of drug interactions 26
Pharmacokinetic interactions 26
Pharmacodynamic interactions 28
Review questions 30

CHAPTER 2: AUTONOMIC PHARMACOLOGY

Part 1: Basic information 39


Part 2: Adrenergic agonists 46
Direct acting sympathomimetic drugs 46
Indirect acting sympathomimetic drugs 51
Mixed acting sympathomimetic drugs 52
Part 3: Adrenergic receptor antagonists 53
Alpha adrenergic blockers 53
Beta adrenergic blockers 58
Part 4: Sympathoplegic drugs 62
Centrally acting sympathoplegic drugs 62
Adrenergic neuron blockers 63
Part 5: Parasympathomimetic drugs 64
Direct acting parasympathomimetics 64
Indirect acting parasympathomimetics 67

vii
Part 6: Muscarinic antagonists 71
Part 7: Ganglion blocking drugs 74
Part 8: Neuromuscular blockers 74
Review questions 78

CHAPTER 3: DIURETIC AGENTS AND VOLUME BALANCE

Part 1: Basic information 85


Part 2: Diuretic classes and agents 87
Loop diuretics 88
Thiazide diuretics 89
Potassium sparing diuretics 90
Osmotic diuretics 92
Part 3: Advantages and disadvantages of diuretics in some edematous 94
conditions
Congestive heart failure 94
Chronic kidney disease 94
Liver cirrhosis 95
Lower limb edema due to pregnancy 95
Part 4: Volume depletion and fluid replacement 96
Part 5: Disorders of serum sodium and potassium 97
Part 6: Manipulation of the urine pH 100
Review questions 102

viii
 
 
 
 
 
 
 
 
 
 
 
 
 
 
 
 
 
 
 
 
 
█ Intro
oductory definitions

Medica al pharma acology is a basic c science.. It the science


s deealing with
h small
molecu ules used tot prevent, diagnose , or treat diseases.
d
Clinicaal pharma acology is s the scieence concerned with the raational, sa afe and
effectivve use of drugs
d in hu
umans. It ccombines elements of o basic phharmacoloogy with
clinical medicine e; in other words, it involves the
t complex interacction betweeen the
drug and the pattient.
A drug g is any chemical molecule that can interact with w bodyy systems at the
molecu ular level and produc ce effect.

The drrug-body interaction


ns

Part 1
1: Ph
harmaco
odynamiics (Mec
chanism of drug a
action)

Pharmaacodynam mics is summmarized a


as what a drug does to the b
body; a drug may
produc
ce its effectts through:
■ Interraction witth body co
ontrol systtems (regulatory prote
eins):
(a
a) Receptorrs (b) Ion
n channelss
(c
c) Enzymess (d) Caarrier molec
cules

■ Direc
ct chemica
al or physic
cal mecha
anisms.
■ Interraction with certain metabolic
m p
pathways.

1
A. REC
CEPTORS
S

Receptors: they are protein macrom molecules. When


W they
y combine with a dru ug, they
may be e activated d or blocke
ed.
Ligand d: is any molecule tha at can com
mbine with the recepptors. A ligaand that ac
ctivates
the rec ceptor is called agonist.
a A ligand that
t block
ks the reeceptor is called
antago onist.
y: it is the empathy of
Affinity o the rece
eptor to the ligand. It determinnes the nummber of
receptoors occupied by the drug.

█ Typ
pes of rec
ceptors

■ Ion channel--linked re
eceptors (direct
ligand-gated
d ion channnels):
- The receptor is an ion
n channell consists
s of 5
transsmembran ne subunits
s (α1, α2, β,, γ, δ).
- Bindding of thee agonist to
t the extrracellular part
p of
the rreceptor causes
c opeening of th he channe el for a
speccific ion.
- The response of these re eceptors iss very fast and their duration
d is very shortt.

- Exam
mples:
 N
Nicotinic Ach e ion channnel opens for Na+
d-plate: the
A recepttors in the motor end
ions in ressponse to stimulation
s n by Ach.
 The Gama a aminobuteric acid (GABA) re n the brainn: the ion channel
eceptors in c
-
opens for Cl ions in response to stimulation by GAABA.

■ G-protein-lin
nked rece
eptors:
- The receptor co
onsists of 7 membranne subunits.
- Bind
ding of the
e agonist too the extra
acellular part
of the receptor ca auses ac ctivation of
acellular G--protein.
intra
- Wheen the G-p protein is activated,
a unit
its α subu
bindds to GTP to be pho osphorylateed and bring
stimulatory or inhibitory response.
- Their responsse is slow wer than ion chann nel
receeptors but their
t duration is long
ger.

- Stim
mulatory G-protein
G (Gs) leadss to increa
ase
enyl cyclase enzyme → ↑ cAMP
ade P → activation
of specific proteins s (proteinn kinase es).
Exa
amples of Gs-couple
G ed receptorrs are the β1
and
d β2-adrenergic receptors.

2
- Inhibitory G-protein (G
Gi) leads to
o decrease yclase enzzyme → ↓ cAMP
e adenyl cy c →
inhibition of protein kinases. Exxamples of
o Gi-coup pled recep
ptors are the
t α2-
adreenergic rec
ceptors an
nd M2 musscarinic rec
ceptors.

- Gq--coupled receptorrs: they increase inositol triphosp phate (IP3 3) and


2+
diac
cylglyceroll (DAG). IP3
3 increasess free intrac
cellular Ca . Exampl es of Gq-ccoupled
rece
eptors are the α1-ad drenergic reeceptors, M1M and M3 3 muscarinnic recepto
ors.

■ Tyro
osine kinase (TK)--linked re
eceptors::
- The receptor consists of 2 largee domainss: an
extraacellular hormone-b
h binding doomain and d an
intra
acellular TKK-binding domain c onnected by a
transsmembran ne segment.
- Bindding of the e agonist to the hoormone-binnding
dommain cause es activattion of th
he intrace
ellular
dommain to ac ctivate TK enzyme → activatio on of
seveeral protein
ns known as
a “signalin
ng proteinss”.

- Exam
mples: insu
ulin recepttors.

■ Intra
acellular recepto
ors:
- Theyy are located inside the
t cell eith her in the cytoplasm
c or directlyy on the DNA.
D
- Theyy regulate transcripti
t on of genees in the nuucleus or the mitoch ondria.
- Their agonist must
m enter inside the
e cell to reaach them.
- Theyy have twoo importantt features:
 Their response is slo ow (time iss required for
f synthes sis of new proteins).
 Their effeccts persist for long tiime after the agonistt is removeed.
- Exam
mples: receptors for corticoste
eroids, sexx hormone
es, thyroxinn, etc.

Types
s of drug
g-recepto
or bonds
s

■ The ionic bon nd:


It is an electric
cal attraction betwee
en two opp
posing cha
arges. It
is strong but reeversible.

■ The hydrogen n bond:


It is an attraction betwe
een two h
hydrogen bonds.
b It is weak
and reversible.

■ The covalent bond:


Veryy strong an
nd irreversible.
If oc
ccurred bettween drugg and rece
eptor, the receptor
r be
ecomes
permmanently blocked.
b

3
█ BIOL
LOGICAL RESPONS
SE TO DRU
UG-RECE
EPTOR BIN
NDING
(Dose
e-response relatiionship s
studies)

When a drug com


mbines with a recepto
or, this ma
ay lead to one
o of the following:
■ Agoonist effect: meanss that the drug com mbines with the recceptor andd gives
resp
ponse.
■ Anttagonist effect:
e mea
ans that thhe drug combines with
w the reeceptor bu ut gives
NO response e, and prev
vents the re
eceptor fro
om binding
g to anotheer drug.

▌Agon
nist effe
ect

Accord
ding to the “dose-rresponse relations es”, theree are 2 ty
ship curve ypes of
responses to drugs:

Graded respon
nse Qu
uantal res ponse

- The response is increase


ed - The
T respon
nse does nnot increas
se
prop portionally y to the do
ose of the proportiona
p ally to the agonist bu
ut it is
agon nist e.g. the response e of the he
eart all-or-none
a e responsee e.g. prevvention
to addrenaline. of
o convulsions by anttiepileptic drugs
- It is tthe respon
nse to mos st drugs. - Itt is responnse to few drugs.
- The response could be tested in on ne - The
T respon nse could nnot be testted in
or mmore anima als. one
o animal and mustt be tested d in a
group
g of animals.

Effecttiveness
s and saffety

■ Effic
cacy
- It is tthe ability of a drug to
t produce
e response
e (effect) affter binding
g to the receptor.
- It is measured d by the Emax (the m
maximal re
esponse that a drug g can eliciit at full
conc
centration)):

4
■ Full agonist is the d drug that gives maximal
m rresponse at full
conceentration (aat full occu
upancy).
■ al agonistt is that ag
Partia gonist givees submax
ximal resp
ponse even
n at full
conceentration i..e. never g ives Emax

■ Pote
ency
- ED5 ve Dose) is the dose
50 (Effectiv e of the dru
ug that giv
ves 50% o
of the Emaxx, or it is
the d
dose that gives
g the desired
d effe
ect in 50%
% of a test populationn of subjec
cts.
- A drug that givves ED50 byb smaller doses is described
d as
a “potentt” drug.
- Poteency of dru
ugs is gene erally less clinically importantt than efficcacy becauuse you
can increase the dose of o a less po otent drug to obtain the effect of a moree potent
one (provided that it is not toxic).

■ Safe
ety
- TD5
50 (Toxic Dose)
D is th
he dose o f the drug needed to cause a harmful effect
e in
50%
% of a test population
n of subjec
cts.

- LD5
50 (Lethal Dose) is th
he dose ne
eeded to cause
c deatth in 50% of a test group
g of
anim
mals. It is experiment
e al term tha
at can be determined
d d in animalls.

5
- Therrapeutic in
ndex (TI) LD50/ED5
L 50:
- It is the ratio between
b th
he LD50 an
nd the ED5
50. It is a measure
m off safety; if there is
a larrge differe
ence betweeen the do ose of a drug
d that produces
p tthe desired
d effect
and the dose that
t producces a toxic
c effect, it is said tha
at the drug has a large TI.
- Druggs with higgh TI are more
m safe ffor clinical use, and vice
v versa (e.g. warfa
arin has
arrow TI and requires careful th erapeutic monitoring
a na g).

▌Anta
agonist effect
e

 Anta
agonist is the ligand
d that com
mbines with
h the recep
ptor and d
does not activate
a
it. Itt has no intrinsic activity, bu ut may cause a pha armacolog gical respoonse by
inhibbiting the actions
a of endogenou
e us substan nces or othher drugs.
 If thee antagoniist binds to
o the same e site of thhe agonist on the recceptor, it is
s called
com mpetitive antagonist
a t. If the an
ntagonist binds
b to an
nother sitee on the re
eceptor,
and prevented d the action
n of the aggonist, it is called nonn-competiitive antag gonist.
 Com m may be reversible
mpetitive antagonism
a e or irreve
ersible:
 R e antagonist makess weak bo
Reversible ond with th
he recepto
or so as you
y can
o
overcome the
t block byb giving h high doses
s of the ago
onist, and even you can get
th
he maximaal response in preseence of the
e antagonis mountable effect).
st (i.e. surm
T
The duratio
on of block
k is short b
because the antagonist can be easily wasshed off
th
he recepto
ors.
 Irrreversible nist make s covalen
e antagon nt bond with the recceptor so as you
ccannot oveercome the block o r get the maximal responser b
by increas
sing the
ddose of the
e agonist (ii.e. non-su
urmountable effect). TheT occup
pied recepttors are
ppermanently blocked d, so the d duration off block is long, and the bodyy has to
ssynthesize new recepptors to reg gain the orriginal state
e.

6
Other types of drug anttagonism

■ Che
emical anttagonism: e.g. one acidic dru
ug when added
a to a basic drrug can
causse precipita
ation of ea
ach other’ss
Exammple: the addition of
o gentamyycin (basic drug) to carpenicilli
c in (acidic drug)
d in
the ssame syrin
nge causes
s chemical complex.

■ Physsical antaagonism: antagonissm betwee


en two
drug
gs carrying opposite charges.
c
Exammple: pro otamine is
s used ffor treatm ment of
hepa dose because protam
arin overd mine carries +ve
charrge while heparin
h ca
arries –ve charge. One
O mg
of prrotamine can
c neutrallize 100 un
nits of hepa
arin.

■ Phys siological antagonism: antaggonism be etween


two drugs producing opposite effe
ects by acttivation
of diifferent rec
ceptors.
Exammple: adre
enaline is the physio ological an
ntagonist of
o histaminne becausse while
hista
amine cauuses hypo otension a and bronchoconstric ction throuugh activa
ation of
hista
amine H1 receptors,, adrenalin ne causes hypertens sion and bronchodiilatation
throu ation of adrrenergic α & β recepttors respec
ugh activa ctively.

■ Pharmacokinetic antag gonism: (seee drug intteractions)).


- One drug mayy prevent absorptio
a on of anoth her drug e.g. antacidds ↓ absorption of
iron & aspirin.
- One drug maay increas se metabo olism of another
a drrug e.g. ri fampicin induces
i
hepaatic enzymmes and ↑ metabolism
m m of oral co
ontraceptivve pills.
- One drug mayy ↑ excretion of anoother drug e.g. NaHC CO3 causee alkalinizaation of
e and ↑ exc
urine cretion of acidic
a drug
gs like asppirin.

B. ION
N CHANNE
ELS

How drugs could modulatte ion cha annels?


 Phyysical block: e.g. blocking of Na+
channels by lo ocal anesth
hetics.
 Thee ion chaannel mayy be parrt of the
receeptor e.g. ion
i channe el-linked re
eceptors.
 Thee ion chann nel may be modulatted by G-
prottein linked receptors.
 Ion channels may be modulated
m by intracellular ATPP e.g. ATPPase senssitive K+
channels in the pancre eatic β cellls, rise off intracellu
ular ATP ccauses clo
osure of
+
pancreatic K channels.

7
C. ENZ
ZYMES

How drugs could affect enzymes?


 Thee drug mayy act as a competitivve inhibito or of the enzyme e.g
g. neostigmmine on
cholinesterasee enzyme.
 Thee drug mayy act as irrreversible inhibitor of
o the enzy yme e.g. o
organophos sphates
on ccholinesterrase enzym
me.
 Thee drug mayy act as a fa
alse subst rate for the
e enzyme e.g.
e α-metthyldopa is s a false
substrate for dopa
d arboxylase..
deca
 Thee drug mayy induce orr inhibit hep
patic microosomal enzymes acttivity (see later).

D. CARRIER MO
OLECULES

 Theese are sm
mall protein
n molecule c moleculees across the cell
es that carrry organic
memmbrane whhen they arre too large
e or too po
olar.
 Drug
gs could affect
a carrie
er molecul es by bloc
cking their recognitio
on site.

Part 2
2: Fac
ctors afffecting dose-re
esponse relation
nship

A. FAC
CTORS RE
ELATED TO
O THE DR
RUG

1. Drug
g shape (s
stereoisom
merism):
- Mosst drugss have
multtiple stere eoisomers
(enaantiomers) (e.g. L-
thyrooxin an
nd D-
thyrooxin). The receptor
site is usually sensitive
for one sterreoisomer
and not suittable for
anotther, like the hand
and the glo ove. This
mea ans that on ne isomer
mayy be hundrred times
more e potent than the
otheer. In other instances one isome er is benefficial while the other is toxic.
- This phenomeenon may explain hhow a sin ngle drug could actt as agon nist and
antaagonist (i.ee. partial agonist)
a b ecause many
m drugss are pressent in “ra acemic
mixttures” rath her than asa pure isoomers; or how one isomer is effective and a the
otheer isomer iss toxic.

8
2. Molecular weight (MW):
- Most drugs have MW between 100-1000 Da. Drug particles larger than MW 1000
Da cannot be absorbed or distributed. They should be given parenterally.
- Drug particles larger than MW 1000 Da cannot cross placental barrier.

3. Time of drug administration (Chronopharmacology):


- Many body functions (e.g. liver metabolism, RBF, blood pressure, HR, gastric
emptying time, etc.) have daily circadian rhythm. Some enzymes responsible for
metabolism of drugs are active in the morning or evening.
- Also many diseases (e.g. asthma attacks, myocardial infarction, etc.) are circadian
phase dependent.
- Chronopharmacology is the science dealing with tailoring drug medication
according to the circadian rhythm of the body to get better response and/or to
avoid possible side effects.
- Examples:
- Episodes of acute bronchial asthma are common at night due to circadian
variation of cortisol and other inflammatory mediators, so it is better to give the
anti-asthmatic medications in the evening.
- Blood pressure is at its peak during afternoon, so it is better to give the
antihypertensive medications at morning.

4. Drug cumulation:
Cumulation occurs when the rate of drug administration exceeds the rate of its
elimination (especially in patients with liver or renal disease). Some drugs are
cumulative due to their slow rate of elimination e.g. digoxin.

5. Drug combination:
Drug combination is very common in clinical practice. When two or more drugs
are combined together, one of the following may occur:

a) Summation or addition:
- Summation means that the combined effect of two drugs is equal to the sum
of their individual effects (i.e. 1+1=2). It usually occurs between drugs having
the same mechanism, for example, the use of two simple analgesics together.

b) Synergism and potentiation:


- Synergism means that the combined effect of two drugs is greater than the
sum of their individual effects (i.e. 1+1=3). The two drugs usually have
different mechanisms of action, for example, the use of penicillin with
aminoglycosides to exert bactericidal effect.

9
- Potentiation is similar to synergism but, in potentiation, the effect of one drug
itself is greatly increased by intake of another drug without notable effect (i.e.
1+0=2), for example, Phenobarbitone has no analgesic action but it can
potentiate the analgesic action of aspirin.

c) Antagonism:
One drug abolishes the effect of the other i.e. 1+1=0 (see before).

B. FACTORS RELATED TO THE PATIENT

1. Age, sex, and weight.

2. Pathological status:
Liver or kidney diseases significantly alter the response to drugs due to altered
metabolism. Also the failing heart is more sensitive to digitalis than the normal heart.

3. Pharmacogenetic factors (idiosyncrasy):


It is abnormal response to drugs due to genetic abnormality in drug metabolism.
These are some examples:

▌Examples of heritable conditions causing EXAGGERATED drug response:

a) Pseudocholinestrase deficiency:
Succinylcholine is a neuromuscular blocker metabolized by pseudo-
cholinestrase enzyme. Some individuals with deficient PsChE, when they take
succinylcholine, severe muscle paralysis occurs due to lack of succinylcholine
metabolism, and may lead to death from respiratory paralysis (succinylcholine
apnea).

b) Glucose-6-phosphate dehydrogenase (G6PD) deficiency:


- G6PD is the most common human enzyme defect. G6PD enzyme catalyzes
the reduction of NADP+ into NADPH which maintains glutathione in the RBCs
in its reduced form. Reduced glutathione keeps Hb in the reduced (ferrous)
form and prevent formation of methemoglobin and cell membrane injury
(hemolysis) by oxidizing drugs.
- Individuals with deficiency of G6PD may suffer acute hemolysis if they are
exposed to oxidizing drugs e.g. nitrates, antimalarial drugs, and others.

c) Thiopurine methyltransferase (TPMT) deficiency:


- Thiopurine methyltransferase (TPMT) is an enzyme that methylates thiopurine
anticancer drugs (e.g. 6-mercaptopurine and 6-thioguanine) into less toxic
compounds.

10
- G eficiency in TPMT lea
Genetic de ads to incre
eased conversion of parent thiopurine
d
drugs into more toxic compou unds, leadiing to seveere myelottoxicity an
nd bone
m
marrow supppression which mayy be fatal.
- T
TPMT defiiciency prrevalence is 1:300. Screening for TPM MT deficieency is
n
necessary in patients
s treated byy thiopurin
ne anticanc
cer drugs.

d) Acetylator phenotyp
p es:
M
Many drugs are me etabolized in the liver by ac cetylation (e.g. isoniazid).
Ac
cetylation reaction is under g genetic co ontrol and
d people ccan be classified
ac
ccording to
o their rate
e of acetyla
ation into rapid
r and slow acetyylators:
- In
n rapid ac
cetylators: excessive
e isoniazid toxic mettabolites aaccumulate
e in the
liver causin
ng hepatoc cellular nec
crosis.
- In
n slow acetylators s: isoniazzid accum mulates in
p
peripheral tissues causing
c peeripheral neuropathy
d
due to inteerference with
w pyrido oxine metaabolism, (so
p
pyridoxine “vit B6” iss added to o isoniazid therapy to
p
prevent neu urotoxicity
y).
- S
Some drug gs that are metabolize ed by acettylation can
c
cause systtemic lupus erythem matosis-like
e syndrome
(S
SLE) in slo
ow acetylattors (see bbox).

▌Exa
amples off heritable
e condition
ns causing
g DECREA
ASED drug
g respons
se:

a) Resistance
e to couma
arin (warfa
arin) antic
coagulants
s:
- In
n normal individuals, warfarin anticoagu
ulant acts by
b inhibitinng the enzzyme vit
K epoxide reductase
r ble for redu
responsib uction of th
he oxidized
d vit K (inac
ctive) to
itts reduced
d form (active).
- S
Some indivviduals havve another variant of this enzymme making g them nee eding 20
times the usual
u dose of coumarrin to get the response.

b) R
Resistance
e to vit D (v
vit D-resisstant ricke
ets):
C
Children witth vit D-res
sistant rickkets need huge
h doses
s of vit D to
o be treate
ed.

c) Re
esistance
e to mydria
atics:
Dark eyes are
a genetically less re
esponsive to the effect of mydrriatics.

4. Hypo
oreactivity
y to drugs
s:
(Tolera
ance; tach
hyphylaxiss; drug res
sistance)

Tolerance mean ns progressive decre ease in drug respon nse with suuccessive admin-
istration. The sam
me respons se could b
be obtained d by highe
er doses. Itt occurs ovver long
period.. Tachyphy ylaxis is an acute typ
pe of tolerance that occurs
o verry rapidly.

11
Mechanism of tolerance:
 Pharmacodynamic tolerance: may occur due to:
 Receptor desensitization: prolonged exposure to the drug leads to slow
conformational changes in the receptors by which the receptor shape
becomes no longer fitted well with the drug.
 Receptor down-regulation: prolonged exposure to the drug leads to decrease
number of the functional receptors.
 Exhaustion of mediators: e.g. depletion of catecholamines by amphetamine.

 Pharmacokinetic tolerance:
 Due to ↑ metabolic degradation of a drug by induction of hepatic enzymes
e.g. with chronic administration of ethanol.

 Behavioral tolerance:
 It occurs by a drug independent learning of the brain how to actively
overcome a certain drug-induced effect through practice e.g. with
psychoactive drugs.

5. Hyperreactivity to drugs:
(Rebound and withdrawal effect)

Rebound effect: is recurring of symptoms in exaggerated form when a drug is


suddenly stopped after a long period of administration.

Mechanism: prolonged administration of the antagonist leads to up-regulation


(increase number) of receptors. When the antagonist is suddenly stopped, severe
reaction occurs e.g. severe tachycardia and arrhythmia occurs after sudden stopping
of beta-blockers.

Withdrawal effect (syndrome) is recurring of symptoms in exaggerated form plus


addition of new symptoms when a drug is suddenly stopped e.g. withdrawal effects
that occur after sudden stopping of opioids in opioid addicts.

N.B. Some examples of drugs should not be stopped suddenly:

Drug Sudden withdrawal can lead to:


Beta-blockers : Severe tachycardia, arrhythmia, and even myocardial infarction.
Clonidine : Severe hypertension (hypertensive crisis).
Cimetidine : Severe hyperacidity and even peptic ulceration.
Corticosteroids : Acute Addisonian crisis.
Morphine : Withdrawal symptoms (see CNS).
Warfarin : Thrombotic catastrophes

12
Part 3: Cllinical pharmac
cokinetic
cs

Definittion: it is th
he journey of the dru g inside th
he body. It includes 4 processe
es:

Abssorption ution
Distribu m
Metabolism
M Excreetion

█ ABS
SORPTION
N OF DRUG
GS

Definittion: it is th
he passage
e of drug f rom the sitte of administration tto the plassma.
The ma ain routes s of administration: o
oral, sublin
ngual, recta
al, inhalatio
on, injection, etc.

Factorrs affectiing drug absorptio


a on:

A. Facttors relate
ed to the drug
d

- Mollecular sizee: small mo olecules arre absorbe


ed than large molecuules.
- Dosse: absorpttion increa ases with in
ncreasing the dose (up to limit)).
- Drug formulattions: e.g. sustained--release ta ablets are slow
s in abssorption.
- Loccal effects of the drugg: e.g. druggs producing VC ↓ thheir own ab bsorption.
- Drug combina v C ↑ abssorption off iron.
ation: e.g. vit
- Lipid solubilityy, drug ion
nization, an
nd the pKaa of the dru
ug.

B. Facttors relate
ed to the absorbing
a g surface:

- Rouute of administration: i.v. route is the fastest while rectal


r is thee slowest.
- Inte
egrity of the
e absorbinng surface: may ↑ or ↓ absorptio on.
- Loccal blood flow: ischemmia ↓ abso orption.
- Speecific facto
ors: e.g. ap
poferritin syystem for iron, etc.

The pK
Ka and drug ioniz
zation

Princip
ples
- Ionized (polarr; charged d) drugs a are poorly absorbed d,
while unionized (non-p polar, non--charged) drugs are e
more absorbe ed.
- Mosst drugs are a weak acids or b bases. Theey become e
ionizzed or non--ionized ac
ccording to the pH aroound them.
- Acid dic drugs (e.g.
( aspirin) are morre ionized in alkaline pH and vicce versa.
- Bassic drugs (ee.g. amphe etamine) a re more ionized in ac cidic pH annd vice verrsa.
- pKa
a of a drug
g: is the pH at whic h 50% of the drug is ionized and 50% is non-
ioniized. (W
Where p = inverse log
g; Ka = association/d
dissociatio
on constant).

13
Examp
ple of pH variation
v and
a drug k
kinetics with
w aspirin
n:

Aspirin is an acid
dic drug; its
s pKa = 3.5
5
The pHH of the sto
omach is 1.5 Th e pH of the intestine
e is 8.5

►Wheen aspirin is put in th he stomac ch:


 Asppirin is acid
dic drug annd become es more abbsorbable in acidic p
pH.
 Logg (Unionize ed /Ionized)* = pKa – pH = 3.5 – 1.5 = 2 (log
( 2 =1022 ).
 Thiss means th hat the rattio of union
nized: ionized = 100/1 (or accuurately 0.9
99 parts
are absorbed and 0.01 parts are n non-absorbbed).

►Wheen aspirin is put in thhe intestin


ne:
 Asp
pirin is acid
dic drug annd become es less abssorbable in
n alkaline p
pH.
 Log
g (Unionize ed/Ionized)* = pKa – ppH = 3.5 – 8.5 = – 5 (log -5 =1 0–5 ).
 Thiss means thhat the ratio of union
nized/ionize
ed = 1/1000000 (or acccurately 0.00001
0
partts are absoorbed and 0.99999 p parts are non-absorb bed).

The above rule applies only to a


*N.B. T acidic drugs
s like aspirrin. For bassic drugs, the
t ratio
would b
be reversed
d.

►Ion ttrapping of
o aspirin:
In the stomach, aspirin is more abssorbable in nto
stomacch cells buut once entered the cells, the pH
changees from 1.5 outside to 7.4 insside the cell.
So asppirin becom mes ionize
ed inside tthe cells and
a
can’t diffuse outsside them again
a → ga
astric ulcer.

al significa
Clinica ance of pK
Ka
 Know
wing the site of drug
g absorptio
on from the
e GIT (see principles)).
 Treaatment of drug
d toxicitty:
- Tooxicity witth acidic drugs
d (e.g . aspirin) could be treated byy alkaliniza
ation of
urrine, which
h renders this drug m more ionize ed in urine and less reeabsorbab
ble.
- Tooxicity with basic drrugs (e.g. aamphetam mine) couldd be treateed by acidiification
off urine, which renderrs this drugg more ionized in urinne and lesss reabsorb
bable.
 Ion ttrapping in
n breast milk:
- Thhe pH of the
t breast milk is 7 i.e. it is coonsidered acidic in rrelation to plasma
(p
pH 7.4).
- Basic drugss (with pKa
a > 7.2) ten
nd to be ionized, and thus trappped, insidee breast
m
milk more thhan acidic
c drugs; heence, the milk/plasm
m a ratio (M//P ratio) would be
hiigh.

14
█ DIST
TRIBUTIO
ON OF DRU
UGS

Sites o
of drug disstribution
 Plassma: 3 liters
 Extrracellular water:
w 9 liters
 Intraacellular water:
w 29
2 liters

▌Volum
me of dis
stribution
n (Vd)

Definittion: The apparent


a volume
v of water into
o which
the druug is distributed in the body after distrribution
equilibrrium.

Calcula
ation:
Total amount
a off the drug in the body y
Vd
d = ————————————— —————————— ————— L
Plassma conc of the drugg (after dis
stribution equilibrium)
e )

Clinica
al significa
ance:
 Deteermination of the site
e of drug d on e.g.:
distributio
- A total Vd < 5 L: means that the e drug is confined
c to
o the vascuular compartment
an
nd can be removed by b dialysiss.
- A total Vd 5-15 L: mea ans that thhe drug is restricted
r to
t the ECF F.
- A total Vd > 41 L: me eans that tthe drug is s highly bo
ound to tisssue prote
eins and
ca
annot be reemoved by y dialysis.
 Calcculation of the total amount o of drug in the body y by singlee measuremment of
plasma concen ntration (from the eq quation).
 Calcculation off the loading dosse (LD) needed to attain a desired plasma
conccentration (Cp): LD = Vdd x Cp.
 Calcculation of drug cleaarance:

▌Binding of dru
ugs to pla
asma pro
oteins

 Mosst drugs wh
hen introdu
uced into tthe body are
a bound tot plasma proteins.
 Albu
umin: the most
m impo
ortant plasmma protein
n and it ca
an bind –vve or +ve charged
c
drug
gs.

Clinica
al significa
ance:
 The pharmaco
ological efffect of the
e drug is related only
y to its fre
ee part no
ot to its
bounnd part (th
he bound part
p acts o
only as a re
eservoir fro
om which tthe drug is
s slowly
relea
ased).

15
 Bind
ding of drugs to plasm
ma protein
ns prolong
gs their effe
ects.
 Wheen the drugg has high
h plasma p nding (e.g. 99% for warfarin), the
protein bin t free
part that exertts the phaarmacologiic effect is s 1%. Anyy small dissplacement of the
boun nd part by another drug
d (say fo
or example e another1% is displlaced) can lead to
drammatic toxic
city (double
es the amo ount of the free part in plasma)..
 Man ny disease e states (e.g. chron nic liver disease, pregnancy, renal failuure) can
affec
ct the level of albumiin and the nature of plasma pro oteins, thuus causing serious
probblems with some drugs.

█▌EXC
CRETION AND ELIM
MINATION
N OF DRUG
GS

ation of dru
Elimina ugs may fo
ollow one o
of 2 proces
sses (orderrs):

First-ord
der elimin
nation Zero-order elim
mination

- Occcurs to mo ost drugs. - Occurs


O to limited nuumber of drugs.
- Connstant ratioo (%) of th
he drug is - Constant
C amount
a off the drug is
i
elim
minated per unit time i.e. the ratte of eliminated
e per unit ti me i.e. the
e rate of
elim
mination is proportional to plassma elimination
e n is not pro oportional to
conncentrationn. The higher the plasma
p concentratio on. A familiar
conncentrationn, the greatter the rate
e of example
e is
s ethanol, cconcentrattions of
elim
mination. which
w decline at a co onstant ratte of
approxima
a ately 15 mg g/100 mL/h h.
- Elim
mination do oes not deepend on - Elimination
E n depends on satura able
satuurable enz zyme system. enzyme
e sy
ystem.
- Thee t1/2 of the drug is co
onstant. - The
T t1/2 of the drug iss not consttant.
- Drug cumulation is not common
c - Drug
D cumu ulation is ccommon

Examp ated by zerro-order: prednisolon


ples of drugs elimina p ne, theophyylline.

N.B. S
Some drug gs are elim
minated b
by first-ord
der elimina
ation in low
w doses and by
zero-orrder elimination in hig
gh doses e.g. aspirin and phenytoin.

16
Clinica
al significa
ance of ze
ero-order
elimina
ation:
 Mod
dest chan
nge in dru
ug dose may
prodduce unexxpected toxxicity.
 Elim
mination off drugs or attainmennt of
Cpsss takes lo
ong time.
 Chaanges in drug form mulation may
prodduce adveerse effects
s.
 Drug cumulattion and innteractionss are
commmon.

▌Elimination half-life (t1/2)

Definittion: It is the time taken forr the


concen ntration of a drug in n blood to
o fall
half to its originall value.

Calcula
ation:
 From
m the plasm ntration ve rsus
ma concen
time
e curve.
 From
m the equaation:

Clinica
al significa
ance:
 Deteermination of inter-d
dosage intterval: dru
ugs are giv
ven every t1/2 to avo
oid wide
flucttuations off the peakk level (the highest plasma con ncentrationn of the drug) and
trouugh level (the lowest plasma co oncentratio on).
 Time e-course of drug accumulat
a tion: if a drug
d is started as a constant infusion,
i
the CCp will accuumulate to approach ssteady-statte after 4-5 5 t1/2.
 Time e-course of drug elimination
e n: If a drug g is stopped after aan infusion, the Cp
will d
decline to re
each comp plete eliminaation after 4-5 t1/2.
 Drug gs having long t1/2 could be give en once daily to imp prove patieent compliiance.

▌Stead
dy-state plasma concentra
c ation (Cps
ss)

Definittion: the steady le evel of d drug in plasma achieved whhen the rate of
adminisstration eq
quals the ra
ate of elim ination.

The rule of 5:
 Thee Cpss is re
eached aftter 4-5 t1/2.

17
 If w
we changedd the dose, the new CCpss is reaached afte
er 4-5 t1/2.
 If do
osing stop
ps, complete eliminattion of drug
g from plasma occurrs after 4-5
5 t1/2.

18
▌Therrapeutic drug
d mon
nitoring (T
TDM)

Definittion: monittoring of se
erum drug concentra ations to optimize druug therapy
y.
 Serum drug samples
s are
a usuallyy taken wh hen the drrug has reeached the e CPSS
(e.g
g. at the tro
ough level, just beforre the next dose).
 TDM M can be done by monitoring g drug effect rather than con centration e.g. in
warrfarin theraapy, TDM is a monitoring the INR (see blood
s done via d).

Clinica
al significa
ance:
 To a
avoid toxiicity in the
e followingg situation ns:
- Drugs withh a low ‘the erapeutic i ndex’ e.g. lithium, diigoxin, andd warfarin.
- PPresence of disease e states (e..g. liver or renal dysffunction) thhat can afffect the
d
drug’s pha
armacokine
etics.
 To improve efficacy
e of drugs ha
aving pharrmacokinettic problem
ms e.g. ph
henytoin
and
d other drugs with no
on-linear kinetics.
 Diffferentiatio en drug ressistance an
on betwee nd patient non-comp
pliance.

▌Clearance as a channe
el of elim
mination

Definittion: plasm
ma clearan
nce of a su
ubstance means
m the
e volume o
of plasma cleared
from th
his substannce per min
nute.

Calcula
ation:

Clinica
al significa
ance of renal cleara
ance:
If the drug is clea
ared by the kidney, cle earance caan help to determine whether th his drug
is eliminated by reenal filtrattion or sec
cretion: a drug
d that is
s eliminateed only by filtration
f
cannot exceed 12 27 ml/min. If clearanc ce > 127 ml/min
m → th
he drug is eeliminated also by
tubularr secretion.

Routess of eliminnation:
 Kidn ney (the major
m route)).
 Bilee and liver.
 Lunngs, intestine, milk, saliva and ssweat.

Clinica
al importance of kno
owing the
e route of eliminatio
e on:
 Help
p to adjustt the dose to avoid c
cumulation.
 Avo
oid drugs eliminated
e by a disea
ased organ
n.
 Targ
geting theerapy: e.g g. drugs eliminatedd by the lung couuld be used as
exp
pectorants..

19
█ MET
TABOLISM
M OF DRUGS (biotra
ansformattion)

 e liver is the major site of drug metabolism but


The b other organs ca an also
mettabolize drrugs e.g. kidney, lung
gs, and adrenal gland ds.
 Manny lipid soluble drugs must bee converted d into a wa ble form (p
ater-solub polar) to
be e
excreted.
 Som
me drugs area not me etabolized a
at all and excreted
e unchanged (hard dru
ugs).
 Mettabolism of
o drugs may
m lead tto:
- Conversion of activ
ve drug in
nto inactiv olites → terrmination of drug
ve metabo
effect.
- Conversion of activ ve drug in nto active metabolittes → prol ongation of drug
effect e.g. codeine (a
active drug
g) is metab
bolized to morphine
m ((active pro
oduct).
- Conversion of inactive drug in nto active metabolites (prodru ugs) e.g. enalapril
e
(inactive drug) is mettabolized tto enalaprilat (active metabolitee).
- Conversion of non--toxic dru ug into toxxic metab
bolites (e.g
g. paracetamol is
converted into the to
oxic produ ct N-acety
ylbenzoquinone).

Bioch
hemical reactions
r s involve
ed in dru
ug metab
bolism

The drug must enter


e phasse I of che emical rea
actions be excreted as water--soluble
compoound. If thee drug is not
n liable tto convers sion into water-solub
w ble compo ound by
phase I, it must enter
e phase II to incrrease solub
bility and enhance
e el imination.

▌Phas
se I reac
ctions

- Pha
ase I reactions include oxidation,
duction, an
red nd hydroly
ysis.
- Enzzymes cataalyzing pha
ase I react ions includ
de
cyto
ochrome P450, ald dehyde a and alcoh hol
deh
hydrogenasse, deam minases, esterase es,
amiidases, and
d epoxide hydratase es.
- Thee majority of phase I reactionss is done by b
the cytochro ome P450 (CYP45 50) enzym me
systtem locate ed primarily inside mmembranou us
vesicles (micrrosomes) on o the surrface of th he
smo ooth endoplasmiic retic
culum of
pareenchymal liver cells s. CYP450 0 activity is
also
o present in otherr tissue e e.g. kidneey,
testtis, ovariess and GIT.

20
- Although this class has more than 50 enzymes, six of them metabolize 90% of
drugs. The most important subfamily is CYP3A4 which is responsible for
metabolism of over 50% of drugs.
- Genetic polymorphism of several clinically important CYP450 enzymes is a
source of variability of drug metabolism in humans.
- Drugs may be metabolized by only one CYP450 enzyme (e.g. metoprolol by
CYP2D6) or by multiple enzymes (e.g. warfarin).
- Some drugs and environmental substances can induce (increase activity) or
inhibit certain CYP450 enzymes leading to significant drug interactions.
- Other examples of non-microsomal oxidation include xanthine oxidase
(converts xanthine to uric acid) and monoamine oxidase (MAO) (oxidizes
catecholamines and serotonin). Only the microsomal enzymes are subjected to
induction or inhibition by drugs.

Microsomal enzyme induction Microsomal enzyme inhibition

 Microsomal inducers ↑ rate of  Microsomal inhibitors ↓ rate of


metabolism of some drugs leading to metabolism of some drugs leading to
↓ their serum levels and therapeutic ↑ their serum levels and toxicity.
failure.  Enzyme inhibition can occur after
 Induction usually requires prolonged short period of exposure to the
exposure to the inducing drug. inhibiting drug.
 Examples of inducing agents:  Examples of inhibiting agents:
phenytoin, phenobarbitone, macrolide antibiotics (e.g.
carbamazepine, rifampicin, smoking, erythromycin), ciprofloxacin,
chronic alcohol intake, St John's cimetidine, ketoconazole, ritonavir,
Wort, grapefruit juice.
 Clinical examples:  Clinical examples:
- Rifampicin accelerates metabolism of - Ciprofloxacin inhibits metabolism of
contraceptive pills leading to failure of warfarin (anticoagulant) leading to
contraception. accumulation of warfarin and
- Phenytoin accelerates metabolism of bleeding.
cyclosporine-A leading to graft - Erythromycin inhibits metabolism of
rejection. theophylline leading to toxicity of
theophylline (cardiac arrhythmia).

▌Phase II reactions (conjugation)

- It involve coupling of a drug or its metabolite to water-soluble substrate (usually


glucuronic acid) to form water-soluble conjugate.
- Glucuronyl transferase is a set of enzymes that is responsible for the majority
of phase II reactions. This set of enzymes is also located inside liver

21
miccrosomes and is the e only phasse II reactio
on that is inducible b
by drugs and
a is a
posssible site of drug in
nteractionss e.g. phen nobarbital induces gglucuronida
ation of
thyrroid hormoone and reduces theiir plasma levels.
- Som me glucuroonide conjjugates seecreted in bile can be
b hydrolyyzed by in ntestinal
bac cteria and the free drug
d can bbe reabsorbed again (enteroheepatic circu
ulation),
thiss can exten
nd the actio
on of somee drugs.
- Oth
her examples of non njugation reactions include sulphate
n-glucuro unide con s
con
njugation (steroids), glycine conjugatio
on (salicylic acid), and gluttathione
con
njugation (e
ethacrynic acid).

▌Firstt-pass me
etabolism stemic elimination
m (pre-sys n)

Definittion: metaabolism off drugs a at the sitee of administration before re eaching


system he liver aft er oral administratio
mic circulattion e.g. th on, the lung
g after inh
halation,
the skin
n after topical adminnistration, e
etc.

Hepatiic first-pas
ss metabo
olism:
 Com
mplete: lido ocaine.
 Parttial: propraanolol, morphine,
nitro
oglycerine
 Non ne: atenolo
ol and mon
nonitrates

How to
o avoid?
- By iincreasing the dose of the drugg.
- By ggiving the drug throu
ugh other rroutes e.g. sublingua
al, inhalatio
on, or i.v.

▌Bioav
vailability
y

Definittion: it is the
t fraction of the d
drug becom me availab ble for sysstemic effe
ect after
adminisstration. The bioavailability of d
drugs given i.v. is 100%.

22
Factors affecting bioavailability:
 Factors affecting absorption.
 Factors affecting metabolism.
 First-pass metabolism.

Part 4: Adverse drug reactions (ADR)

An ADR is any response to a drug which is noxious, unintended, and occurs at


doses used in man for prophylaxis, diagnosis or therapy.

Predisposing factors:
 Multiple drug therapy.
 Extremes of age: due to age related changes in pharmacokinetics and dynamics.
 Associated disease: e.g. impaired renal or hepatic function.
 Genetics: can affect the pharmacokinetics.

Classification:

▌Type A (Augmented):

These reactions are predictable from the known pharmacology of the drug. They
may result from an exaggerated response (e.g. hypotension from an
antihypertensive) or non-specificity (e.g. anticholinergic effects with tricyclic
antidepressants).

Prevention
 Take a careful history for predisposing factors.
 Use the smallest dose of the drug adequate for the desired effect.
 Adjust dosage to therapeutic end-points, e.g. blood pressure or INR.
 Adjust dosage to optimum plasma concentrations, e.g. digoxin.
 Adjust dosage in relation to renal function, hepatic function, or other drugs.

▌Type B (Bizarre):

These are less common, less predictable, and may be severe. Examples are:
 Immunologic: penicillin allergy
 Genetic: haemolysis in G6PD deficiency
 Disease: amoxycillin rash in glandular fever
 Idiosyncratic: malignant hyperpyrexia in anesthesia.

Prevention
 Take a careful drug history, especially of allergies

23
 Fam
mily historyy: allergies or genetic
c disease
 Avo
oid drugs susceptibl
s e to ADRss in particular diseas
se states, e.g. cloza
apine in
bon
ne marrow depressio
on.

Type A (Augmen
nted) Ty
ype B (Biz
zarre)
- Pred
dictable - Unpredictable
- Dosee-dependeent - Dose-independent
- High
h incidence
e - Low incid
dence
- Mayy respond to
t dose ad
djustment - Generally
y need to sstop the drrug

█ Drug
g-induced
d liver injjury (DILII)

DILI ac
ccounts forr up to 10%
% of all advverse drug
g reactionss and may be fatal.
It may be classifie
ed into:
 Acccording to time courrse: acute or chronic.
 Acccording to mechanis sm: dose-d dependentt, idiosynchhratic, or im
mmune me ediated
 Acccording to histologic g: hepatoce
cal finding ellular, cho
olestatic, o r mixed picture.

Hepato
ocellular (cytotoxic)
( ) DILI Cholestatic
C c DILI
Featurres: Features:
F
 The e drug or its metaboliites affectss  The drug or its meetabolites affect
a
pareenchymal liver cells leading
l to the bilia
ary canalicuuli leading to
cell necrosis and
a initiatioon of narrowin ng or dest ruction of biliary
infla
ammatory process. passage es.
 It m
may be spo otty, zonal, or diffuse..  Clinically it resemb bles obstruuctive
 Clinnically it ressembles viral hepatittis jaundice e with prurritus and ↑ALP.
A
withh ↑ ALT and AST.

Commmon drugs:: Common


C drugs:
d
Parace
etamol – methyldopa – Chloroprom
C mazine – suulfonylureaas –
amioda
arone – iso
oniazid – va
alproic acid
d oral
o contrac ceptive pil ls – anabo
olic
steroids
s – macrolides
m s – co-amo oxiclav

24
█ ADR on pregnancy

Key facts:

 Fetal birth defects represent 2-3% of all births, the majority of which are related
do drugs.
 Some fetal defects may be impossible to identify, or can be delayed e.g. the use
of diethylstilbesterol (estrogenic compound) during pregnancy is associated with
development of adenocarcinoma of girls’ vagina at teen age.
 Three factors determine the risk of teratogenicity: dose of the drug; duration of
administration; and stage of pregnancy.
 Most drugs with a MW <1000 can cross the placental barrier.
 All drugs should be considered harmful until proven otherwise.

Mechanism of teratogenicity according to pregnancy stage:

 Before implantation: (0-17 day): The effect is all-or-none i.e. either death of the
embryo (abortion) or no effect.

 Early pregnancy: (3-10 weeks):


- It is the most dangerous period because it is period of organogenesis.
- Selective interference can produce characteristic anatomical abnormality e.g.
aminoglycosides cause damage to 8th cranial nerve.

 Late pregnancy:
- Gross anatomical abnormalities are less liable to occur.
- Functional defects rather than anatomical abnormalities can occur especially
in organs having delayed formation e.g. brain, testes, and bone.

Examples of teratogenic drugs:

 ACE inhibitors and angiotensin receptor blockers cause fetal pulmonary and
renal dysfunction.
 Antithyroid drugs can cause fetal goiter and hypothyroidism.
 Tetracycline antibiotics inhibit growth of fetal bones and stain teeth.
 Aminoglycosides cause fetal 8th cranial nerve damage.
 Warfarin can cause fetal intracerebral bleeding.
 NSAIDs cause premature closure of ductus arteriosus.
 Benzodiazepines cause cleft lip and palate.
 Sex hormones can cause inappropriate virilization or feminization.
 Antiepileptic drugs cause neural tube defect (spina bifida).
 Cytotoxic drugs can cause multiple structural damage.

25
The FDA pregnancy categories:

Category Definition Animal Human


Adequate studies in animal and human did
A not show a risk to the fetus either in the  
first or in the late trimesters.
Animal studies did not show risk to the  ?
fetus but there are no adequate studies in
human.
B
or: Animal studies showed a fetal risk, but
adequate studies in human did not show a  
risk to the fetus.
Animal studies showed a risk to the fetus
but there are no adequate studies in  ?
C humans; the benefits from the use of the
drug in pregnant women may by
acceptable despite its potential risks. Benefit > Risk

There is evidence of human fetal risk, but  


D the potential benefit of the drug may
outweigh its potential risk. Benefit > Risk
Studies in animals and humans showed  
evidence of fetal risk. The potential risk of
X
use in pregnant women clearly outweighs
Risk > Benefit
any potential benefit.

Part 5: Principles of drug-drug interactions

Classification:
■ Pharmacokinetics interactions.
■ Pharmacodynamic interactions.

█ PHARMACOKINETIC INTERACTIONS

Drug interactions in vitro:


e.g. antipseudomonal penicillins and aminoglycosides form complexes in the infusion
fluid (see chemotherapy).

26
Drug interactions in vivo:

Absorption

 Formation of complexes:
- Tetracycline forms complexes with Ca2+, Mg2+ and Al3+
- Cholestyramine forms complexes with digitalis and thyroxin.

 Absorption can be blocked:


- Adrenaline ↓ absorption of local anesthetics due to VC.
- Colchicine ↓ absorption of vitamin B12

 Change in intestinal motility:


- Anticholinergic drugs ↓intestinal motility → ↑ absorption of some drugs.
- Prokinetic drugs ↑ intestinal motility → ↓ absorption of some drugs.

 Changes in gastric pH:


- Antacids ↓absorption of salicylates.
- Ketoconazole is poorly absorbed in absence of gastric acidity.

Distribution

- Sulfonamides displace bilirubin from pl pr in premature infants → kernicterus.


- Phenylbutazone displaces warfarin → excessive bleeding.

Metabolism

- Inhibition or induction of microsomal metabolism (see before).


- Inhibition of non-microsomal enzymes:
- MAO inhibitors ↓ metabolism of some drugs e.g. benzodiazepines, serotonin
and norepinephrine.
- Disulfiram inhibits acetaldehyde dehydrogenase enzyme → ↓ metabolism of
acetaldehyde → accumulation of acetaldehyde causes flushing, nausea,
vomiting, and tachycardia.

Excretion

 Reduction in urinary elimination:


- Probenecid ↓ renal excretion of penicillin.
- Quinidine ↓ renal excretion of digoxin.

 Changes in urinary pH:


- Alkalinization of urine (e.g. sodium bicarbonate) ↑ excretion of weak acids
- Acidification of urine (e.g. ammonium chloride) ↑ excretion of weak bases.

27
 Changes in urinary volume:
Diuretics can increase toxicity of some drugs by reducing plasma volume e.g.
thiazide can increase lithium toxicity.

 Stimulation of biliary excretion:


Phenobarbital ↑ biliary excretion of many drugs by increasing both bile flow and
the synthesis of conjugating proteins.

██ PHARMACODYNAMIC INTERACTIONS

 Antagonism: competitive, non-competitive, chemical, physical, etc.


 Synergism: e.g. MAO inhibitors can cause toxic synergism with TCA.
 Potentiation: e.g. ethanol can enhance CNS depression caused by opioids
 Changes in the intracellular or extracellular environment: e.g. diuretic-
induced hypokalemia can ↑ digitalis toxicity.

28
 
29
 
30
Review Questions

Define the following pharmacokinetic parameters:


 Volume of distribution
 pKa of drugs
 Elimination half life
 First-pass metabolism
 Bioavailability

Mention the clinical significance of each of the following:


 Volume of distribution
 pKa of drugs
 Plasma protein binding of drugs
 Elimination half-life
 Zero-order elimination
 Microsomal enzyme induction
 Hepatic conjugation of drugs

Mention the main differences between:


 Reversible and irreversible antagonism.
 Graded response and quantal response.
 First order elimination and zero order elimination.
 Potency and efficacy.
 Physical and physiological antagonism.
 Habituation and addiction.
 Oxidation and conjugation of drugs.

Discuss 2 pharmacogenetic conditions associated with toxic drug response


Discuss 2 pharmacogenetic conditions associated with reduced drug response
Write short account on antagonism between drugs

31
Of each of the following questions, D. Sex hormones act on these types of
select ONE BEST answer: receptors
E. Corticosteroids act on these types of
1. A drug may act by all the following receptors
mechanisms EXCEPT:
A. Interaction with protein 5. The following statements are true
macromolecules embedded in the cell for graded dose-response relationship
membranes EXCEPT:
B. Interaction with cell membrane ion A. It is the response to most drugs
channels B. The response is directly proportional
C. Interaction with intracellular enzymes to drug concentration (linear relation)
D. Interaction with cell membrane C. It could be tested in one animal
phospholipids D. It can be used for comparing the
E. Interaction with gene functions potencies and efficacies of drugs
E. It can be used for calculation of the
2. Ion-channel-linked receptors (direct LD50 of drugs
ligand-gated ion channels) are
characterized by: 6. The following statements are true
A. They are the type of receptors for quantal dose-response relationship
principally present in autonomic EXCEPT:
ganglia and skeletal ms motor end A. It is the response to anticonvulsant
plate and antiarrhythmic drugs
B. They are the type of receptors B. The response to the drug is not
principally present in vascular directly proportional to drug
endothelium concentration (all-or-none)
C. They are rosette-shaped structures C. It could be tested in one animal
consist of 7 membrane subunits D. It helps in calculation of the ED50 and
D. Their response is slower than other LD50 of drugs
receptors E. It helps in estimation of the degree of
E. Activation of these receptors leads to drug safety
activation of a second messenger
7. When a drug has a steep dose-
3. Which of the following is classified response curve, this means:
as belonging to the tyrosine kinase A. The drug is lethal
family of receptors:
B. The drug is expensive
A. GABA receptors
C. The drug is efficacious
B. β-Adrenergic receptors
D. The drug is safe
C. Insulin receptors
E. Minimal change in the dose can lead
D. Nicotinic acetylcholine receptors to dramatic effect.
E. Hydrocortisone receptors
8. The following statements are true
4. All the following are true for for drug’s therapeutic index EXCEPT:
intracellular (DNA-linked) receptors A. It is the relation between the lethal
EXCEPT: dose in 50% of animals to the curative
A. They regulate transcription of genes dose in 50% of them
inside the nucleus B. The lower the TI, the safer will be the
B. Their response is very fast but drug.
persists for long time C. It should be done to any drug before
C. Their agonists must enter inside the it’s being approved for human use
cell to reach them inside the nucleus

32
D. For theoretically useful drugs, it must E. Is described as addition if the action
be greater than 1 of one drug abolishes the effects of
E. It could be applied in animal testing another

9. The following is true for competitive 13. A drug may interact with ion
antagonism: channels by all of the following
A. It never occurs with enzymes mechanisms EXCEPT:
B. Is the same as physiological A. The drug may change the ion channel
antagonism structure
C. The agonist can never abolish the B. The drug may block the channel
effect of the antagonist physically
D. Is best exemplified by the use of C. The drug may change an intracellular
neostigmine to treat curare toxicity ATP on which the channel depends
E. Best described as non-surmountable D. The ion channel may be part of ion
process channel-linked receptors
E. The ion channel may be modulated by
10. A drug is said to be reversible G-protein linked receptors
antagonist when:
A. It blocks the receptors by making 14. Failure of the patient to breath after
covalent bonds with them surgical operation may be due to:
B. The duration of blockade is too long A. Pseudocholinestrase deficiency
C. Increasing the dose of the agonist will B. Methemoglobin reductase deficiency
reverse the block C. G-6-PD deficiency
D. The response curve of the agonist in D. Vitamin K epoxide reductase
presence of this drug is not parallel to deficiency
that of the agonist alone E. Monoamine oxidase deficiency
E. Termination of the drug effect dep-
ends on synthesis of new receptors 15. Hemolysis that may occur with
sulfonamides therapy may be due to:
11. The interaction that may occur A. Pseudocholinestrase deficiency
between acidic and basic drugs is B. Methemoglobin reductase deficiency
called: C. G-6-PD deficiency
A. Chemical antagonism D. Vitamin K epoxide reductase
B. Physical antagonism deficiency
C. Physiological antagonism E. Monoamine oxidase deficiency
D. Biological antagonism
E. Receptor antagonism 16. Severe myelosuppression following
6-mercaptopurine therapy is most likely
12. The following is true for interactions due to:
between drugs: A. Pseudocholinestrase deficiency
A. Is not harmful if occurred between B. Methemoglobin reductase deficiency
drugs having steep dose-response C. G-6-PD deficiency
curves D. Vitamin K epoxide reductase
B. Is not harmful if occurred between deficiency
drugs having narrow therapeutic E. Thiopurine methyltransferase
ratios deficiency
C. Is not harmful if occurred between
drugs undergoing zero-order kinetics 17. Hepatic toxicity that may
D. May lead to valuable therapeutic accompany isoniazide therapy may be
effects due to:

33
A. Defective oxidation reaction D. Phenobarbital (pKa = 7.4)
B. Defective conjugation reaction E. Propranolol (pKa = 9.4)
C. Defective deamination reaction
D. Slow acetylation reaction 22. The following statements are true
E. Rapid acetylation reaction for Vd of drugs EXCEPT:
A. It can exceed the volume of water in
18. Failure of some children with rickets the body
to respond to therapeutic doses of B. Drugs with large Vd can be removed
vitamin D is most likely to be due to: by dialysis
A. Differences in sex C. Would be expected to be 5L if the
B. Differences in body weight drug is confined to the blood.
C. Genetic variation D. Highly lipid-soluble drugs would be
expected to have large Vd
D. Tolerance
E. Intolerance E. It can help in the calculation of the
total amount of the drug in the body
19. The following are true for overshot
phenomenon (drug intolerance)
23. The plasma half-life (t1/2) of drugs:
EXCEPT: A. Is expressed as the percentage that
remains ½ hour after administration
A. It occurs due to down-regulation of
receptors B. Will be short if the drug gets into the
enterohepatic circulation
B. It occurs after sudden stoppage of
some drugs given for long time C. Cannot be calculated if the drug is
excreted through the bile
C. It may lead to serious withdrawal
effects D. Is constant for drugs having zero-
order elimination
D. It can be avoided by gradual
cessation of drugs E. Can be prolonged by slowing the rate
of drug elimination
E. It is best exemplified by occurrence of
severe tachycardia after sudden
stopping of beta blockers. 24. The bioavailability of a drug:
A. Is defined as the actual blood
20. The following statements are true concentration required to produce a
for pKa of drugs EXCEPT: pharmacological effect
A. Ionized drugs are poorly absorbed B. Will be unaffected by changes in
while unionized drugs are more formulation
absorbed C. May be affected by liver damage
B. Ionization of most drugs depends on D. Must be 100% for a drug given by
the pH of the medium around them mouth and is completely absorbed
C. pKa of drugs can help knowing the E. Is a term applied only to oral
site of drug absorption. administration
D. Acidic drugs become more
absorbable in alkaline pH 25. All the following are phase I
E. Basic drugs become more biotransformation reactions EXCEPT:
reabsorbable in alkaline urine A. Sulfate conjugation
B. Xanthine oxidation
21. Which of the following drugs will be C. Nitroreduction
absorbed to the LEAST extent in the D. Ester hydrolysis
stomach: E. Oxidative deamination
A. Ampicillin (pKa = 2.5)
B. Aspirin (pKa = 3.5)
C. Warfarin (pKa = 5.0)

34
26. Metabolism (biotransformation) of E. Drug X will have a shorter duration of
drugs can lead to all the following action than drug Y because less of
results EXCEPT: drug X is present for a given effect.
A. Conversion of active compound into
inactive metabolites 30. Which of the following terms best
B. Conversion of active compound into describes the antagonism of
active metabolites leukotriene’s bronchoconstrictor effect
C. Conversion of inactive compound into (mediated at leukotriene receptors) by
active metabolites terbutaline (acting at adrenoceptors) in
D. Conversion of non-toxic compound a patient with asthma?
into toxic metabolites A. Pharmacologic antagonist.
E. Conversion of water-soluble B. Partial agonist.
compound into lipid-soluble C. Physiologic antagonist.
metabolites D. Chemical antagonist.
E. Noncompetitive antagonist.
27. All the following statements are true
for First-order kinetics EXCEPT: 31. Which of the following provides
A. Apply to most drugs in clinical use information about the variation in
B. Apply to salicylate (aspirin) sensitivity to the drug within the
metabolism within small dose. population studied?
C. The concentration versus time curve A. Maximal efficacy.
is non-linear. B. Therapeutic index.
D. The rate of elimination depends on C. Drug potency.
plasma concentration of the drug D. Graded dose-response curve.
E. Steady state plasma concentration E. Quantal dose-response curve.
can be reached after 5 half lives
32. Which of the following provides
28. All the following statements are true information about the largest response
for zero-order kinetics EXCEPT: a drug can produce, regardless of
A. Elimination rate is independent of the dose?
dose A. Drug potency.
B. Elimination depends on saturable B. Maximal efficacy.
enzyme system
C. Mechanism of receptor action.
C. Plasma concentration of the drug D. Therapeutic index.
cannot be expected at any time
E. Therapeutic window.
D. The t1/2 of the drug is not constant
E. There is no fear from drug cumulation 33. A pro-drug is:
or interactions
A. The prototype member of a class of
drugs.
29. Drugs X and Y have the same
mechanism of diuretic action. Drug X in B. The oldest member of a class of drugs
a dose of 5mg produces the same C. An inactive drug that is transformed in
magnitude of diuresis as 500 mg of the body to an active metabolite.
drug Y. This suggests that: D. A drug that is stored in the body
A. Drug Y is less efficacious than drug X tissues and is then gradually released
in the circulation.
B. Drug X is about 100 times more
potent than drug Y. E. Ionized drug trapped in breast milk.
C. Toxicity of drug X is less than that of
drug Y. 34. If the rate of infusion of a drug were
doubled, what response in the steady
D. Drug X is a safer drug than drug Y.

35
state concentration would be E. About 99%
expected?
A. Remain unchanged 40. Concerning the renal excretion of
B. Doubled drugs:
C. Increase 50% A. Drugs that are ionized in the renal
D. Decrease 50% tubules are more likely to undergo
E. Decrease 100% passive reabsorption.
B. Low MW drugs are much more likely
35. Half-life of a drug may be helpful to to be actively secreted than filtered.
determine: C. Only the fraction of the drug that is
A. Elimination of the drug unbound (free) to plasma proteins is
filtered by the glomerulus.
B. Level of absorption
C. Rate of absorption through the GIT
D. Decreasing urinary pH enhance
excretion of weakly acidic drugs.
D. Time to reach the steady state
E. Renal clearance cannot exceed the
E. Distribution into body systems. GFR (125 ml/min).

36. What determines the degree of 41. In which of the following cases
movement of a drug between body could a graded dose-response curve be
compartments? constructed?
A. Partition constant A. Prevention of convulsions
B. Degree of ionization B. Prevention of arrhythmias
C. pH C. Reduction of death
D. Molecular size D. Reduction of fever
E. All of the above E. Relief of insomnia
37. For intravenous (IV) dosages, what 42. Which of the following can be used
is the bioavailability assumed to be? as a relative indicator of the margin of
A. 0% safety of a drug?
B. 25% A. T.I.
C. 50% B. LD50
D. 75% C. ED50
E. 100% D. EC50
E. TD50
38. Which of the following can produce
a therapeutic response? A drug that is: 43. Flurazepam has a pKa of 8.2. What
A. Bound to plasma albumin percentage of flurazepam will be
B. Concentrated in the bile ionized at a urine pH of 5.2?
C. Concentrated in the urine A. 0.1%
D. Not absorbed from the GI tract B. 1.0%
E. Unbound to plasma proteins C. 50%
D. 99%
39. Aspirin is a weak organic acid with E. 99.9%
a pKa of 3.5. What percentage of a
given dose will be in the lipid-soluble 44. Which route of administration is
form at a stomach pH of 1.5? most likely to subject a drug to first
A. About 1% pass metabolism?
B. About 10% A. Intravenous
C. About 50% B. Sublingual
D. About 90% C. Oral

36
D. Inhalation E. Phase I metabolism of Drug A will
E. Intramuscular increase its intracellular access and
actions
45. If a drug was given by a constant
infusion rate, which of the following 48. The FDA assigns the letters A, B, C,
factors determines how long it will take D, and X to drugs approved for human
for the drug to reach a steady-state use. To which of the following does this
concentration (Cpss) in the blood? classification apply?
A. Apparent volume of distribution A. Amount of dosage reduction needed
B. Bioavailability as serum creatinine clearances fall
C. Clearance B. Fetal risk when given to pregnant
women
D. Half-life
E. Infusion rate (mg of drug/min) C. Amount of dosage reduction needed
in presence of liver dysfunction
46. Which of the following best D. Relative margins of safety/therapeutic
index
describes what the term
“tachyphylaxis” means? E. The number of unlabeled uses for a
drug
A. An increase in the rate of the
response, for example, an increase of
the rate of muscle contraction 49. Which effect may lead to toxic
reactions when a drug is taken
B. Immediate hypersensitivity reactions
continuously or repeatedly?
(i.e., anaphylaxis)
C. Prompt conformational changes of the A. Refractoriness
receptor such that agonists, but not B. Cumulative effect
antagonists, are able to bind and C. Tolerance
cause a response D. Tachyphylaxis
D. Quick and progressive rises in the E. Intolerance
intensity of drug response, with
repeated administration, even when 50. Tolerance and drug resistance can
the doses are unchanged be a consequence of:
E. Rapid development of tolerance to the A. Change in receptors, loss of them or
drug’s effects exhaustion of mediators
B. Increased receptor sensitivity
47. Drug A undergoes a series of Phase C. Decreased metabolic degradation
I metabolic reactions before being D. Decreased renal tubular secretion
eliminated. Which of the following
statements best describes the
E. Activation of a drug after hepatic first-
pass
characteristics of Drug A, or the role of
Phase I reactions in its metabolism?
51. If two drugs with the same effect,
A. Complete metabolism of Drug A by taken together, produce an effect that
Phase I will yield products that are
is equal in magnitude to the sum of the
less likely to undergo renal tubular
effects of the drugs given individually, it
reabsorption
is called as:
B. Drug A is a very polar substance
A. Antagonism
C. Drug A will be biologically inactive
until it is metabolized B. Potentiation
D. Phase I metabolism of Drug A C. Synergism
involves conjugation with glucuronic D. Additive effect
acid or sulfate E. Supersensitivity

37
52. All of the following statements
about efficacy and potency are true
EXCEPT:
A. Efficacy is usually a more important
clinical consideration than potency
B. Efficacy is the maximum effect of a
drug
C. Potent drugs usually given in small
dose.
D. Potency is a comparative measure,
refers to the different doses of two
drugs that are needed to produce the
same effect
E. The ED50 is a measure of drug’s
efficacy

Answers

1D 11 A 21 E 31 E 41 D
2A 12 D 22 B 32 B 42 A
3C 13 A 23 E 33 C 43 E
4B 14 A 24 C 34 B 44 C
5E 15 C 25 A 35 D 45 D
6C 16 E 26 E 36 E 46 E
7E 17 E 27 C 37 E 47 A
8B 18 C 28 E 38 E 48 B
9D 19 A 29 B 39 E 49 B
10 C 20 D 30 C 40 C 50 A
51 D
52 E

38
 
 
 
 
 
 
 
 
 
 
 
 
 
 
 
 
 
 
 
 
 
Part 1
1: Basiic inform
mation

The au utonomic nerv-


ous sy ystem co ontrols
involunntary ac
ctivity
(Fig 1, ttable 1).

Sympa athetic nervous


systemm (SNS)
‒ Sho ort prega anglio-
nic axons orig ginate
from
m thoracic c and
lum
mbar areas of the
spinnal cord and
synapse in ganglia
locaated closse to
the spinal corrd.
‒ Thee adrenal me-
dulla is consiidered
ammodified ganglion and d is innerva
ated by sym
mpathetic fibers.
‒ Theermoregulaatory swea at glands aare anatommically sym mpathetic, but the postgan-
glio
onic nerve fibers
f relea
ase acetylccholine (AC
Ch) (i.e. sym
mpathetic cholinergic).

Parasyympathetic c nervouss system (PPNS)


‒ Lon ng pregangglionic axo
ons originaate from cranial
c and
d sacral arreas of thee spinal
cordd and synnapse in ganglia
g loc
cated clos se to or within
w the innervatedd organ
(with few exce
eptions).
‒ Sho ort postgan
nglionic ax
xons innervvate many tissues an
nd organs aas the SNS S.
‒ Parasympatheetic innervvation preddominates over symp pathetic in nervation of most
orgaans exceppt blood veessels (ha ve only sym
mpathetic supply).

39
Entericc nervous system (E ENS)
‒ Thee ENS is co onsidered the
t third ddivision of the
t ANS.
‒ It iss a collectiion of neurons insid e the wall of the GIT
T that conntrols the motility,
m
exo
ocrine and endocrine secretionss of the GI tract.
‒ Nerrve terminaals contain peptides and purine es as neuro
otransmitteers.
‒ Thiss system functions
f in
ndependen ntly of the CNS and is modulaated by bo oth SNS
andd PNS.

omatic nerrvous systtem contro


The so ols volunta
ary activity::
‒ Lonng axons originate in the spinal cord and directly
d innervate ske cles (no
eletal musc
gan
nglia).
‒ Nerrve termin nals in thet neurromuscula ar junctio on releasse Ach as the
neu
urotransmittter.

Neuro
otransmiitters of the ANS
S

1. Norrepinephriine and epinephrin


ne

They arre catecho


olamines, having
h cate
echol nucleu
us.

Biosyn
nthesis of catechola amines:
‒ In n
nerve endin ngs, tyrosinne is hydro b tyrosine hydroxylasse to form (dopa);
oxylated by
dop
pa is then n decarboxylated to o form doopamine which
w is hhydroxylated into
nore
epinephrinne inside sttorage vessicles.
‒ In ccertain areaas of the brain and in the ad drenal
meddulla, noreepinephrinee is meth hylated by
y N-
metthyltransferrase to form epineph hrine.

Storagge and rele


ease:
‒ Norrepinephrinne is stored in vessicles in nerve
n
term
minals.
‒ Norrepinephrinne also exxists in a non-vesicular
cyto
oplasmic pool
p that is released
d by indirrectly
acting symp pathomime etics (e.gg., tyrammine,
amp phetaminee).

Termin
nation:
■ R e (80%): mainly in the
Re-uptake e form of:
‒ Neuronal uptake (in
nto neuron al cytoplas sm).
‒ Granular uptake (into storagee vesicles).

■ M
Metabolism
m (18-20%
%):
Figure
F 2. Syynthesis and termination
of
o norepinephhrine

40
‒ Monoam
mine oxidas enzyme: metabolizes
se (MAO) e m s norepine phrine in neuronal
n
cytoplasmm.
 MAO--A: present in the braain and pe eripheral tis
ssues (e.g. liver & inte
estine).
 MAO--B: presen nt mainly iin the Bra
ain and mo ore active on dopam mine. It
has litttle effect on
o norepinnephrine annd seroton nin.
‒ Catecho
ol-O-methy
yl transfe OMT): metabolizes nnorepineph
erase (CO hrine in
synaptic space.

N.B. Th he end-prooduct of catecholami


c ine metabo olism is
vanillyl mandelic acid
a (VMA).. The norm
mal urinary level of
VMA iss 4-8 mg//day. Higher levels indicate tu umor in
suprare enal medulla
a (pheochroomocytom ma, Fig 3).

Clin
nical
corrrelates:
Deppression is s asso-
ciateed with dec creased
activvity of NA and/or
sero
otonin at th he level
of ssynapse. Tricyclic
T
antiddepressant drugs ac ct by inhibbition of neuronal
n
uptaake of NA and
a seroton nin while MA
AO inhibitin
ng drugs
act bby inhibition
n of their metabolism.
m . Both mechanisms
lead to accum mulation off NA and serotonin at the
synaaptic levels.

Park kinsonism is associated witth deficie ency of


dopaamine in the
t negrostriatal pathhway. Inhib
bition of
MAO O-B enzym me (selegiline) and COMT enzyme
(tolc apone) lea
ads to acccumulation of dopamine and
imprrovement off bradykinesia.

2. Ace
etylcholine
e (ACh)
‒ ACh
h is synthe
esized in nerve termiinals from acetyl co--A and cho
oline. Synthesized
AChh is stored in vesicless in nerve terminal.
‒ Bottulinum tooxin blocks s Ach releaase and ca auses skele
etal musclee paralysis
s.
‒ Thee main fatee of ACh iss rapid hyddrolysis byy cholinestterase (ChhE) enzyme; there
are two isofoorms:

True Ch
hE Pseudo ChE
C
‒ Present in CNS, ganglia, NMJ.
N ‒ Presennt in plasmaa and liver.
‒ Speccific for Ach
h. ‒ Not sp
pecific for A
Ach
‒ Defic
ciency is fattal. ‒ Deficie
ency is Nott fatal
‒ Regeenerates in 2-3 monthss. ‒ Regennerates in 2 -3 weeks.

41
Clinica
al correla
ates:
Congen
nital PsChE
E deficienc
cy (succinyylcholine ap
pnea):
Succinyylcholine is a neuromuscular blo ocker that is
i metabolized by
PsChE enzyme. Some S indiv
viduals havve congen nital deficie
ency of
PsChE, when theyy take succinylcholine tto produce muscle relaxation
before surgery, severe
s musscle paralyysis occurs
s due to lack of
succinyylcholine metabolism.
m Death maay occur from
f paralyysis of
respirattory musc cles. Urgent blood transfusio on and artificial
a
respirattion may be
e required.

3. Co-ttransmittters
A number of Non n-adrenerrgic-Non-c cholinergiic (NANC) transmitteers may be e found
ociation wiith NA or Ach
in asso A in the e autonomic nerve teerminals. T They are re
eleased
with th
he primaryy transmittter to plaay a regu ulatory function. Exxamples include:
i
neuroppeptide Y; encephalin
e ; histamine
e; 5HT; ATP
P; PGs; and
d nitric oxid
de (NO).

42
Table 1. Distribution and functions of autonomic receptors.

SYMPATHETIC PARASYMPATHETIC
Tissue
R Effect R Effect

Heart β1 ↑ all cardiac properties M2 ↓ SA node activity and AV


(tachycardia, ↑A-V conduction, conduction (NOT atrial
↑contractility, etc) conduction)
α1 VC of most BV VD of most BV (through release
Blood of EDRF). N.B. Most vascular M3
β2 VD of skeletal muscle BVs, and M3*
vessels receptors are non-innervated
coronary artery.
Bronchi
Smooth ms β2 Relaxation (Bronchodilatation) M3 Contraction (Bronchoconstriction)
Glands
α1 ↓ Bronchial secretion M3 ↑ Bronchial secretion
GIT
Wall α, β2 Relaxation (↓ motility) M3 Contraction (↑ motility)
Sphincters α1 Contraction M3 Relaxation
Salivary gld α1 ↑ enzyme secretion (viscid saliva) M3 ↑ water secretion (salivation)
Liver β2 Glycogenolysis −
Stomach HCl − M1 ↑ HCl secretion
U bladder
Detrusor ms β2 Relaxation M3 Contraction
Sphincter α1 Contraction (urine retention) M3 Relaxation (urine flow)
Uterus β2 Relaxation −
α1 Contraction
♂ organs α1 Ejaculation M3 Erection
Kidney β1 ↑ Renin secretion −
Skeletal ms β2 Tremors and enhancement of −
neuromuscular transmission
Eye
Iris ms α1 Pupil dilatation (mydriasis) M3 Pupil constriction (miosis)
Ciliary ms β2 Relaxation (distant vision) M3 Contraction (near vision)
IOP β2 ↑ aq humor secretion (↑ IOP) M3 ↑ aq humor drainage (↓ IOP)
Lacrimal gld − M3 ↑ lacrimal secretion
Sweat gld α1 ↑ sympathetic sweating M3 ↑ Thermoregulatory sweating
(forehead & palms) (cholinergic sweating)
Fat cells β3 Lipolysis −
Mast cells β2* ↓ histamine release −
Plasma K+ β2 Decrease plasma K+ −
Nerve α2 ↓ NA release −
terminals
β2 ↑ NA release

* = Non-innervated receptors i.e. receptors are found in the organ but have no autonomic nerve
supply. They can respond only to circulating or administered agonists.
EDFR = endothelial derived relaxing factor = nitric oxide (NO).

43
44
Table 2. Summary of adrenergic receptors

α1 α2 β1 β2 β3
2nd msngr Gq (↑ IP3 & ↑ DAG) →↑ Ca2+ Gi (↓ cAMP) Gs (↑ cAMP) Gs (↑ cAMP) Gs (↑cAMP)
Sites and 1. VC of most bl vessels 1. Presynaptic nerve 1. ↑ all cardiac 1. Presynaptic nerve endings (↑ ↑ lipolysis
function (α1A) endings (↓ NA properties NA release) (adipose
2. Contraction of all release) 2. ↑ renin release 2. Central: ↑ central sympathetic tissue)
sphincters (GIT, urinary). 2. Central: ↓ central (kidney) outflow
3. Contraction of dilator sympathetic outflow 3. VD of sk ms bl vessels and
pupillae ms (mydriasis) 3. Relaxation of GIT & coronary artery
4. Contraction of uterus UB walls 4. Bronchodilatation
5. Relaxation of GIT & UB 5. Relaxation of GIT & UB walls
walls 6. Relaxation of uterus
6. Adrenergic sweating 7. Skeletal muscle tremors
(forehead & palm)
8. ↑ aqueous humor secretion
9. ↑ liver glycogenolysis
10.↓ plasma K+

Sel. agonist Phenylepherine Clonidine Dobutamine Salbutamol


Selective Prazosin Yohimbine Atenolol Butoxamine (not used clinically)
antagonist
Non-selec Adrenaline Ephedrine
agonist
Non-selec Phenoxybenzamine Ergot alkaloids Propranolol Timolol
antagonist

■ β4 and β5 are also present but still under investigation.


■ In most smooth muscles, the α1 receptors mediate contraction through activation of Ca2+ dependent myosin light chain kinase but in the
GIT smooth muscles, they mediate relaxation through hyperpolarization caused by opening of Ca2+ dependent K+ channels.
■ α1 receptors have 3 subtypes, A, B, and D; α2 receptors have three subtypes: A, B, and C.
Table 3. Summary of cholinergic receptors

M1 M2 M3 Nn Nm
2nd msngr Gq (↑ IP3 & ↑ DAG) Gi (↓cAMP) Gq (↑ IP3 & ↑ DAG) →↑ Ca2+ Ion channel Ion channel
→↑ Ca2+

Sites and 1. CNS ↓↓ SAN activity and 1. VD of most BV through synthesis of All autonomic NMJ → skeletal
function AV conduction (not endothelial-derived relaxing factor (EDRF) ganglia and muscle
2. Stomach → ↑
atrial conduction) → ↓ blood pressure. adrenal medulla contraction
HCL secretion
2. Contraction of all wall smooth muscles
(bronchi, GIT, UB) and relaxation of all
sphincters.
3. ↑↑ all body secretions (sweating,
salivation, lacrimation, etc).
4. Eye: → miosis & ciliary muscle contraction
(accommodation for near vision).

Selective
Pyrenzepine Gallamine --------- Trimetaphan d-tubocurarine
antagonist
Non-selec
Acetylcholine
agonist
Non-selec
Atropine - hyoscine
antagonist

N.B.
 M4 and M5 are also present in the CNS.
 M1 – M3 – M5 are linked to Gq (↑ IP3 & ↑ DAG).
 M2 – M4 are linked to Gi (↓ cAMP)

45
Part 2
2: Adrrenergic
c agonis
sts (Sym
mpathom
mimetics
s)

Actions and che


emical stru
ucture

■ Thesse drugs act


a either directly
d or indirectly (by release of norep
pinephrine stored),
to aactivate adrenergic
a c receptorrs and mimicm the
e effects of endo ogenous
cate
echolaminees.
■ Not all sympa athomimetiic drugs a activate the adrenergic recepttors by the same
degrree; some e drugs have hig her affinitty toward ds certainn receptor class
(sele
ectivity) depending
d on the u ltrastructu ure of the
ese recepttors; howeever, in
large
e doses, this
t selecttivity is los
st.
■ Chemically, these drugs may conttain catech
hol nucleus
s, (i.e.
cate
echolamin nes) or not (non-cate
echolaminnes).
■ Cateecholamine es (e.g. epinep phrine, norepinephrine,
isoprroterenol, and dopammine) are ccharacteriized by:
 Thhey are noot absorbe nd cannott pass the BBB.
ed orally an Figure 5. Catechol
C
 Thhey have very
v short duration due to rappid inactiv
vation nucleus
byy MAO and d COMT.

█ DIRE
ECT- ACT
TING SYMP
PATHOMIIMETIC DR
RUGS

1. Epinephrine
e (Adrenaline)

Chemistry and pharmaco


p kinetics
 Epin
nephrine is a natural catechola
amine neurrotransmitter. Syntheetic epinep
phrine is
poorly absorb bed from thhe GI tractt and does s BBB to aany extent.
s not pass
 It is administereed parente
erally (i.m. and s.c.). The bsorption iss slow due to local
T s.c. ab
VC.
 Nebulized form ms (inhalation) are alsso available.
 It is metabolizeed rapidly by COMT and MAO. Metabolittes are exccreted in urine.

Mecha
anism and pharmac
cological e
effects
 Epin
nephrine acctivates alll α and β-a adrenocepptors.
 β recceptors meediate their effects thhrough inc
crease intra
acellular cA
AMP.
 α1 reeceptors mediate
m the
eir effects through in
ncrease intracellular IIP3 and DA
AG.

CVS  Increase rate


r (chro notropic effect)
e and
d force (innotropic efffect) of
(The major the cardia
ac muscle (β1); it ma
ay precipittate anginaa in patien
nts with
action) coronary insufficienc
cy.
 Increases c pressurre due to
s systolic t positive
ve inotrop
pic and

46
chronotropic effectts (β1), and
a decreeases dia astolic prressure
(because VD of skeeletal musc cle blood vessels
v (β22) overcommes the
VC producced by α1 rreceptors in skin and splanchnicc vascular beds).
 At high do
oses, VC ((α1) of all vascular beds
b dominates leading
pred
to increas
se both syystolic and diastolic BP.
B
 Increase coronary blood flo
ow due to
o increaseed cardiac work
ation of me
(accumula etabolites),, and β2 stimulation ((VD).
Respirratory  Relaxation
n of bronch
hial smooth muscle (β2).
(
system
m  Decrease bronchial secretionss (α1).
Eye  Mydriasis due to conntraction of
o dilator pupillae muuscle (α1)
Metabo olic  Hyperglyccemia due to stimulattion of heppatic glyco
ogenolysis (β2).
effects
s  Lipolysis and
a increasse free fattty acids in blood.

Therap es (emerge
peutic use ency condittions)

■ Anaphylactic shock:
 It is a life-tthreatening g conditio on (acute
hyp persensitivity reactio on) resultting from
masssive release of histamin ne from
inflaammatory cells in res sponse to exposure
to allergic substance e (e.g. penicillin).
Histtamine cau uses severe hypoten nsion and
bronchoconsttriction by b its eeffect on
histtamine (H1) receptors s.
 Inje
ection of ep pinephrine immediate ely dilates the bronchi (β2), deccreases brronchial
secretions (α α1), and elevates
e B
BP (VC); so, epine ephrine iss considerred the
“phhysiologica e” of hista
al antidote amine as itt can reverrse all its eeffects by actions
on d different re
eceptors.
 It iss given 0.5 5 ml (1:100
00) i.m. (id eally in the
e lateral th
high musclles) and could be
repe eated /5 min
m if no ressponse.

■ Acutte bronch hospasm: within m minutes affter s.c. administrat


a ation, epinephrine
indu
uces bronc chodilation
n (β2) and decrease airway ed dema (α1);; however, short-
actin
ng selectivve β2 agonists could be used with
w fewer CVSC side eeffects.

■ Carddiac arres st: early i.v.


i epinep
phrine adm ministration, during cardiopulmonary
resu
uscitation (CPR),
( cann restore c
cardiac activity (β1) and impro ove vascullar tone
colla
apse (α1). Recomme
R nded dosee is 1 ml off 1:1000 i.v
v. or 10 ml 1:10000 i..v.

■ Withh local anesthetics


a s: diluted concentrrations of epinephrrine (1:100000) or
noreepinephrinee are som
metimes ad dded to loc
cal anesthhetics to p
produce lo
ocal VC;
this will prolo
ong the duration off the anes ue to ↓ aabsorption of the
sthetic (du
anessthetic) and
d reduce bleeding.
b

47
Adverse effects
 Cerebral hemorrhage: due to marked elevation of BP.
 Anginal pain from excessive cardiac work and strain.
 Cardiac arrhythmia: especially if given with digoxin, or if given i.v. (i.v. epinephrine
can produce fatal ventricular fibrillation).
 Acute pulmonary edema: due to increase both systemic and pulmonary
pressures, this cause acute rise of intrapulmonary hydrostatic pressure with
hydrostatic flux of fluid.

Contraindications

 Presence of hypertension: as it may cause cerebral hemorrhage.


 Ischemic heart disease.
 Cardiac arrhythmia or with digitalis: epinephrine can worsen the arrhythmia
and even precipitate ventricular fibrillation.
 Cardiac outflow obstruction (e.g. severe aortic stenosis, severe pulmonary
hypertension, hypertrophic obstructive cardiomyopathy). In these conditions,
epinephrine will increase contractility of the cardiac muscle against a narrow
outlet, so it can precipitate acute pump failure.
 Thyrotoxicosis: because patients with thyrotoxicosis have increased
sympathetic overactivity (tachycardia, tremors, anxiety, etc) due to increased
sensitivity of β receptors.

2. Norepinephrine

 It activates α (mainly) and β1- The baroreceptor reflex


receptors leading to increase both  It is a physiological reflex
systolic and diastolic BP with reflex concerned with regulation of BP.
bradycardia.  Sudden increase of BP →
 It has little activity on β2-receptors. activation of stretch receptors in
 It is used as VC (by slow i.v. infusion) the carotid sinus and aortic arch →
in acute hypotensive states. afferent impulses through the
vagus to the CVS centers in the
medulla→ decrease HR and
3. Dopamine vascular tone. Hypotension has
opposite effect.
 It is a natural catecholamine given by i.v.  These receptors begin to respond
infusion because it has very short at pressures ≥170 mm Hg.
duration of action (2 min).  In chronic hypertension, this set
 In low doses: stimulates dopamine point is shifted to higher level.
D1 receptors in renal and mesenteric  Baroreflex is blocked by atropine.
vascular beds leading to VD and
increase renal and hepatic blood flow.

48
 In intermediate doses: stimulates cardiac β1 receptors leading to increase
contractility and COP.
 In large doses: stimulates vascular α1 receptors leading to VC and ↑↑ BP.

Therapeutic uses
■ Shock states:
 Shock is a complex state of hypotension associated with impaired tissue
perfusion of the vital organs (brain, liver, kidney, etc.).
 Dopamine, given by continuous i.v. infusion, restores adequate tissue perfusion
by increasing COP (β1), and increasing renal blood flow (RBF) and glomerular
filtration rate (GFR; D1). In high doses, it also stimulates vascular α1 receptors
leading to improvement of vascular tone collapse and elevation of BP.
 N.B. If vasopressors (e.g. noradrenaline) are given alone, they will elevate BP but
aggravate tissue ischemia due to VC.

4. β-adrenoceptor agonists:

a. Selective β1 agonist: Dobutamine

 Dobutamine is a synthetic catecholamine that is related to dopamine. It is


administered by i.v. infusion because of its short duration (2 min).
 It activates mainly cardiac β1-receptors with no effect on dopamine receptors
leading to increase COP with little or no vascular effects.
 It is given by continuous i.v. infusion in cardiogenic shock (a complication of left
ventricular infarction), to reverse myocardial depression and increase COP (β1).

Dopamine Dobutamine
‒ Natural catecholamine ‒ Synthetic catecholamine
‒ Stimulates D > β1 > α1 ‒ Stimulates β1 only
‒ Used for treatment of most cases ‒ Used mainly for treatment of
of shock cardiogenic shock

b. Selective β2 agonists:
Salbutamol, terbutaline, salmetrol, formoterol, ritodrine

 They are synthetic non-catecholamines.


 Salbutamol and terbutaline have short duration – salmetrol and formoterol have long
duration.
 They have greater selectivity at β2 receptors leading to relaxation of bronchial
and uterine smooth muscles, and decrease peripheral resistance (VD).
 In high doses, selectivity on β2 receptors is lost, leading tachycardia and even
arrhythmia (β1).

49
Therapeutic uses:
 Treatment of bronchial asthma (see respiratory for more details).
 Ritodrine is commonly used to induce uterine relaxation and delay preterm labor.

Adverse effects
 High doses can cause hypotension (from VD), tachycardia and arrhythmia due to
loss of selectivity.
 Tremors: β2 receptors in skeletal muscles neuromuscular junction facilitate
neuromuscular transmission and induce tremors.

c. Non-selective β agonist: Isoproterenol (Isoprenaline)

 Synthetic catecholamine that predominantly stimulates both β1& β2 receptors.


 It increases HR and contractility, and relax bronchial smooth muscles.
 It is used as a bronchodilator; however, its nonselectivity is one of the major
drawbacks that makes it rarely used therapeutically.

5. α-adrenoceptor agonists:

a. Phenylephrine, methoxamine, and medodrine

 They are non-catecholamines having long duration of action.


 They selectively stimulate α1-receptors leading to VC, and increased both
systolic and diastolic pressures with reflex bradycardia.
 They are used as vasopressors to correct hypotension.
 They could be used locally as eye or nose drops to produce VC and relieve
congestion (i.e. nasal decongestants).

b. Xylometazoline and oxymetazoline

 These drugs stimulate both α1 and α2 receptors but with slight selectivity toward
α1 receptors.
 They are primarily used locally as eye or nose drops to produce VC (nasal
decongestants).

Adverse effects and precautions


‒ Sufficient concentrations may be absorbed systemically and produce severe
hypertension (even stroke); so they must be avoided in hypertensive patients.
‒ Prolonged and continuous use (> 3 weeks) may lead to atrophy of the nasal
mucosa (due to VC); so they should be used for the minimum duration.
‒ Repeated local application leads to rebound severe congestion, so they must be
used with the minimal dose and for the shortest duration.

50
c. Clonidine

 It is a centrally acting α2 agonist leading to decrease central sympathetic


outflow and blood pressure (see page 55).

d. Tizanidine

 It is another centrally acting α2 agonist (congener of clonidine) with greater effect


on presynaptic α2 in the spinal cord, so it inhibits neurotransmission and
reduces muscle spasm with minimal effect on blood pressure.
 It is used as skeletal muscle relaxant in various spastic conditions such as
multiple sclerosis, back pain, and spine injuries.

6. Dopamine receptor agonists: Fenoldopam

 It stimulates peripheral dopamine (D1) receptors in renal and mesenteric arteries,


leading to VD and decrease peripheral resistance.
 It is used parenterally as a rapid-acting vasodilator to treat emergency
hypertension in hospitalized patients.

█ INDIRECT- ACTING SYMPATHOMIMETIC DRUGS

1. Amphetamine and its derivatives

 Amphetamine and its derivatives are


indirect-acting adrenergic drugs. They Attention deficit
mediates their primarily action by blocking hyperactivity disorder
neuronal uptake of dopamine and (ADHD)
norepinephrine and promoting their It is a neuropsychiatric disorder
release from store sites in the CNS and appears after the age 6 years.
synapses. The child is hyperkinetic and
 Amphetamine stimulates the entire CNS, lacks attention in addition to
cortex, brainstem, and medulla. This leads poor school performance.
to increased alertness, decreased fatigue, The amphetamine derivative
depressed appetite, and insomnia. methylphenidate is a CNS
stimulant that can improve
attention spans allowing better
Therapeutic uses
function in school, and reduces
‒ Methylphenidate is an amphetamine hyperkinesia associated with this
derivative used in the treatment of syndrome.
attention-deficit/ hyperactivity disorder
(ADHD) of children.
‒ Modafenil is another derivative used for treatment of narcolepsy.

51
Adverse effects
Narcolepsy is a chronic
‒ High doses cause anxiety, seizures, neurological disorder character-
hypertension, chest pain, and life- ized by intermittent, uncontrolla-
threatening arrhythmia. ble episodes of falling asleep
‒ Psychosis, hallucinations, and drug during the daytime. These
dependence. sudden sleep attacks may occur
during any type of activity at any
time of the day.
2. Cocaine

 Cocaine is an alkaloid derived from the coca plant. It is widely abused as a


recreational stimulant.
 It inhibits neuronal uptake of norepinephrine, dopamine, and serotonin leading
to their accumulation in the synaptic spaces with profound CNS stimulation.
 Adverse effects: similar to amphetamine
 Manifestations of cocaine toxicity is managed by benzodiazepines.

█ MIXED-ACTING SYMPATHOMIMETIC DRUGS

Ephedrine

 Ephedrine is a plant alkaloid effective


Chronic orthostatic
orally and, unlike catecholamines,
hypotension
penetrates the brain and can produce
CNS stimulation. On standing, venous return is re-
 Ephedrine acts by both: duced by the effect of gravity. Nor-
mally, BP decrease is prevented by
‒ Release of norepinephrine from nerve
reflex sympathetic activation with
endings.
increased HR, and peripheral VC.
‒ Direct stimulation of α and β
Impairment of autonomic reflexes
receptors (weak and prolonged).
that regulate BP can lead to chronic
 Tolerance develops to ephedrine after orthostatic hypotension. This might
continuous administration. be caused by drugs that impair
autonomic function (e.g., tricyclic
Therapeutic uses antidepressants and α blockers),
‒ Ephedrine is used as bronchodilator and diabetes, and autonomic neuropa-
thy.
CNS stimulant, but its clinical use is
now declining because of the Drugs that activate α1 receptors
availability of better, more potent agents (e.g. Midodrine) can be used for
this indication. Ephedrine can be
with fewer side effects.
also used (rarely).
‒ Pseudoephedrine is one of the four
isomers of ephedrine. It is present in
many nasal decongestant mixtures.

52
Table 4. Selected therapeutic uses of adrenoceptor agonists (sympathomimetics)

Clinical condition Agonist Receptor


Cardiac arrest Epinephrine β1 – α1
Anaphylactic shock Epinephrine β2 – α1
Shock (most types) Dopamine D1 - β1
Cardiogenic shock Dobutamine β1
Chronic orthostatic hypotension Midodrine, phenylephrine α1
Bronchial asthma Salbutamol, terbutaline, salmetrol β2
Premature uterine contractions Ritodrine β2
Running nose (rhinitis) Oxymetazoline, xylometazoline α1
ADHD Methylphenidate ?
Narcolepsy Modafenil ?

Part 3: Adrenergic receptor antagonists

These drugs interact with either α- or β-adrenoceptors to prevent or reverse the


actions of endogenously released catecholamines or exogenously administered
sympathomimetics.

█ α- ADRENERGIC BLOCKERS

■ Non-selective α-receptor blockers: phenoxybenzamine, phentolamine


■ Selective α1-receptor blockers: prazosin , terazosin, doxazosin
■ Selective α2-receptor blockers: yohimbine
■ The ergot alkaloids.

1. Non-selective α- blockers: Phenoxybenzamine

 Phenoxybenzamine is a noncompetitive, irreversible antagonist at both α1 and


α2 receptors. It binds covalently with α receptors, resulting in long-lasting blockade
(15 –50 h).
 Blockade of α receptors leads to orthostatic hypotension and reflex tachycardia.

Therapeutic uses
■ Management of pheochromocytoma
 Phenoxybenzamine or phentolamine are used for long-term management of
inoperable tumors.

53
 β-receptor antagonists are
often given after α-blockers Pheochromocytoma
to prevent the cardiac effects It is tumor of the adrenal medulla that secretes
of excessive catecholamines excess catecholamines → headache,
(see box). hypertension, palpitations, sweating, and
dyspnea. 10% of the tumors are malignant.
Adverse effects Diagnosis:
 Orthostatic hypotension and  CT scan.
 High levels of VMA in urine
reflex tachycardia.
 Impairment of ejaculation. Treatment:
 Miosis.  Surgical excision of the tumor.
 Combined  and  lockers to block all
adrenergic receptors:
2. Selective α1- blockers: ‒ Phenoxybenzamine 100 mg /day +
Prazosin, terazosin, propranolol 50 mg/day.
doxazosin, tamsulosin ‒ Start first with phenoxybenzamine to
control BP then add -blocker.
 Prazosin is the prototype drug. ‒ If α-blockers are used alone, the
 All of these agents decrease elevated catecholamines will act on
peripheral resistance and unopposed β-receptors leading to severe
lower arterial BP by: palpitations, arrhythmia, etc.
‒ If β-blockers are used alone, the
 α1- receptor blockade.
elevated catecholamines will act on
 Direct VD of both arterial unopposed α-receptors leading to severe
and venous smooth hypertension.
muscles.  Labetalol is a combined  and  lockers
 They cause minimal changes that could be used alone.
in COP, RBF, and the GFR.
 They don’t trigger reflex tachycardia by the same degree as the non-selective
blockers.
 They improve plasma lipid profile and decrease LDL and TGs.
 Doxazosin has the longest duration of action (22 h).

Therapeutic uses
 Treatment of mild-to-moderate hypertension: especially in patients with renal
failure because it does not decrease RBF or GFR.
 Treatment of congestive heart failure because they decrease both the afterload
and preload through combined arteriolo- and veno- dilatation (see CVS).
 Benign prostatic hyperplasia (BPH) and impaired bladder emptying because
blockade of α1 receptors in smooth muscles of the bladder neck and prostate
leads to decrease resistance to urine flow. The old drug prazosin is no longer
recommended for this indication.

54
Tamsulosin is the most commonly used
Salt and water retention
for treatment of BPH because:
induced by BP lowering
‒ It has high affinity for α1A & α1D, the 2 drugs
receptor subtypes responsible for  It occurs as a compensatory
mediating smooth muscle contraction response after long duration of
in prostatic tissue. antihypertensive therapy in the
‒ It has little effect on standing BP form of ankle edema and slight
compared with other α1-blockers. weight gain.
 Hypotension leads to reflex
Adverse effects stimulation of the renin-
angiotensin-aldosterone system
 First dose hypotension (syncope) which causes fluid retention.
‒ Occurs more frequently with prazosin.
 Diuretics are often prescribed
It starts 30-90 min after the first dose. with BP lowering drugs to
‒ It occurs more frequently in salt and minimize this effect.
water depleted patients.
‒ Prevention: start with a small dose at bedtime then increase the dose gradually.
 Fluid retention (salt and water retention): (see box).
 False positive test for antinuclear factor of rheumatoid arthritis.
 α- blockers can worsen incontinence in women with pelvic floor pathology.

3. Selective α2- blockers: Yohimbine

 Selective presynaptic α2-blocker that leads to increase norepinephrine release.


 It is sometimes used as aphrodisiac (enhance sexual desire) without clinical
evidence.

4. The ergot alkaloids

Chemistry and pharmacokinetics

 Ergots are a wide variety of compounds that are produced by the fungus
Claviceps purpurea. These agents have a strong structural similarity to
norepinephrine, dopamine, and serotonin.
 They may be natural or semi-synthetic:

Natural alkaloids Semi-synthetic alkaloids


 Ergotamine  Dihydroergotamine
 Ergometrine  Methylergometrine
 Ergotoxine: a mixture of three other alkaloids. It  Dihydroergotoxin
is very toxic and not used clinically.  Bromocryptine

55
 Ergots may be administered parenterally, rectally, or orally, and vary widely in
their degree and speed of absorption.
 The absorption of ergotamine is increased by caffeine.
 Ergots are extensively metabolized to compounds of varying activity and half-life.

Mechanism and pharmacological effects

 Ergots act as agonist, antagonist or partial agonists at three receptor types: α-,
dopamine, and serotonin receptors.
 The pharmacologic use of ergots is determined by the relative effect of each
member on these receptors.

Drug Receptor Main effect Main uses


Ergotamine Partial agonist at Direct VC with greater Acute migraine
α- and 5-HT effect on cerebral BVs attacks
receptors
Methlyergometrine α-receptors Uterine smooth Postpartum
agonist muscle contraction hemorrhage
Dihydroergotoxin α-receptors VD with greater effect Senile cerebral
antagonist on cerebral BVs insufficiency
Bromocryptine Dopamine agonist Pituitary and ne- Suppress lac-
grostriatal pathway tation and ttt of
parkinsonism

Therapeutic uses

■ Ergotamine: acute migraine


‒ The major effect of ergotamine is cerebral VC by a direct action; it reverses the
rebound VD that is the probable cause of pain.
‒ It should be given at the start of aura because it has slow onset. It is better to
be combined with caffeine because caffeine increases its absorption.
‒ If given before aura (e.g. for prophylaxis), it can induce cerebral VC and
precipitates the acute attack (i.e. contraindicated).
‒ The VC induced by ergotamine is long-lasting and cumulative; therefore, pat-
ients must not take more than 6 mg of the oral preparation for each attack.

■ Methylergometrine: postpartum hemorrhage


‒ It causes prolonged and forceful contraction of uterine smooth muscles.
‒ Ergots should not be used to induce labor. It should be given at the time of
placenta delivery (3rd stage of labor) and never before that. If it is given before
delivery of the placenta, it causes severe spasm of uterine smooth muscles
and retained placenta.

56
■ Brommocriptine e: hyperprrolactinem mia
‒ Bromocriptin ne is a dopamine rec ceptor agoonist that causes
c inhiibition of prolactin
p
seecretion (high prolactin levels c
can induce infertility and
a ameno orrhea in women).
w
‒ It is used to suppre ess norma al lactation
n and as a dopam ine alterna ative in
Paarkinson’ss disease

■ Dihy oxin: used


ydroergoto d as cerebrral vasodila
ator in senile cerebraal insufficie
ency.

Advers
se effects of ergot alkaloids
a
 Nauusea and vomiting
v due to stim ulation of
CTZZ.
 High doses ca ause VC of
o small artterioles of
fing
gers leadinng to coldd hands a and even
ganngrene (erg
gotism).
 VC of coronarry artery with
w anginall pain.
 Uteerine contrraction and
d abortionn if given
duriing pregna
ancy.

█ MIGR
RAINE AND
D SOME DR
RUG TREAT
ATMENTS

ne is severre unilatera
Migrain al periodic headache characterized by:

- The stage of aura:


a occu
urs in 15-3 0% of
casees. Suddenn release of
o serotonin n (and
otheer mediatoors) of unnknown ettiology
caussing VC ofo cerebra al BV → vvisual,
olfac
ctory or au
uditory hallucinationss.
- The stage of pain: prolonged VC leads
accuumulation of waste e metaboliites →
seveere VD → perivascu ular edem a and
seveere headacche.

▌Drugs
s used in the
t acute attack
■ Ergo
otamine and dihydro mine (see before).
oergotam
■ Triptans: sum
matriptan, zolmitript
z an
– Theey are agon
nists at 5-H
HT1D and 5
5-HT1B rece
eptors.
– Activation off 5-HT1D receptorss inhibits inflammation of meningess, pain
tran
nsmission, and release of VD ssubstances citonin genne-related peptide
s e.g. calc
in trigeminal neurons. Activation
A of 5-HT1BB receptorss causes V VC of the dilated
cereebral vesssels. Abou ut 50%–800% of pattients repo ort relief frrom pain within
w 2
houurs after oral administration.

57
– 5-HT1B activity can cause coronary spasm so; these drugs are contraindicated
in patients with ischemic heart disease (IHD).
– Like ergotamine, they are also contraindicated for prophylaxis of migraine or in
combination with ergotamine because severe hypertension and coronary spasm
may occur.

▌Drugs used for prophylaxis


 Propranolol: the most commonly used drug for prophylaxis.
 Ca2+ channel blockers: verapamil (unclear mechanism).
 Clonidine: (unclear mechanism).
 Tricyclic antidepressant (TCA) drugs.

█ β- ADRENERGIC BLOCKERS

The β-receptor–blocking drugs differ in their relative affinities for β1 and β2


receptors; however, the selectivity is dose-related and tends to diminish at higher
doses.

■ Non-selective β-blockers: e.g. propranolol, pindolol, timolol, sotalol, nadolol


■ Cardio-selective β1-blockers: e.g. atenolol, metoprolol, bisoprolol, nebivolol.
■ β-blockers with additional VD action: e.g. dilevalol, carvedilol
N.B. the VD action comes from either: blocking the vascular α1 receptors;
increasing PGE2 and PGI2 synthesis; or by release of endothelial NO.

Chemistry and pharmacokinetics

 Propranolol is the prototype β-adrenoreceptor antagonist.


 β-blockers are absorbed well after oral administration, many have low
bioavailability because of extensive first-pass metabolism.
 Lipophilic β-blockers (e.g. propranolol) can pass readily to the CNS and are
cleared by hepatic metabolism. Hydrophilic β-blockers (e.g. atenolol) have
limited penetration to the CNS and are excreted primarily by the kidney with little
hepatic metabolism.
 β-blockers which undergo hepatic metabolism usually require multiple daily
dosing. Drugs eliminated via the kidney are suitable for once daily administration.

Mechanism and pharmacological effects

CVS effects  They block cardiac β1 receptors and decrease all cardiac
properties (↓ contractility and COP, ↓ A-V conduction
"bradycardia", ↓ excitability, and automaticity).

58
 They block the β2-mediated VD in peripheral vessels leading to ↓
blood flow to most tissues.
 They decrease blood pressure through:
 ↓↓ COP by their –ve inotropic and chronotropic effects.
 ↓↓ renin release from the kidney (β1).
 ↓↓ norepinephrine release and central sympathetic outflow (by
blocking presynaptic β2).
 They cause resetting of baroreceptors to a lower level (see
before).
 Some β-blockers block also vascular α1 receptors.
 Some β-blockers enhance synthesis of vasodilator PGE2 and
PGI2.

Respiratory  β-blockers produce bronchospasm as a result of β2-receptor


blockade; this effect is more significant in asthmatic patients.
 Even selective β1 blockers can interact with β2 receptors in high
doses and produce bronchospasm, especially in asthmatic
patients.

Eye  ↓ IOP by decreasing aqueous humor secretion from the ciliary


epithelium (timolol and betaxolol have excellent effect).
 Sufficient timolol can be absorbed after topical application to
increase airway resistance and decrease HR and contractility.

Metabolic  Inhibition of glycogenolysis in the liver (β2) leading to aggravation


effects of hypoglycemic effect of insulin and other hypoglycemic drugs.
 ↑ plasma K+ (hyperkalemia) in patients with renal failure.
 ↑ plasma triglycerides and ↓ HDL.

CNS  Antianxiety effects.


 Night mares, vivid dreams, and depression.
 Sexual dysfunction through combined central and peripheral
mechanisms.

Skeletal ms  ↓ essential tremors due to blocking of β2 in skeletal muscles.

Other  Propranolol has local anesthetic (membrane stabilizing) action i.e.


specific it can inhibit excitability of the cardiac muscle.
properties  Pindolol is a partial agonist i.e. it doesn’t cause excessive
bradycardia.
 Esmolol is ultrashort acting (t1/2 = 10min) because of extensive
hydrolysis by plasma esterases; it is administered by i.v. infusion to
control arrhythmia during surgery and emergency situations.
 Labetalol blocks β-receptors and α1-receptors (mixed blocker).

59
 Carvedilol has additional antioxidant action.
 Nebivolol is the most selective β1 blocker.

Therapeutic uses

■ Treatment of hypertension: (see the mechanisms above). All beta-blockers,


irrespective of their properties, lower BP to a similar extent.

■ Ischemic heart disease (classic angina and acute MI)


‒ They decrease contractility, HR, and systolic BP, thus decrease myocardial work
and O2 demand (in acute myocardial infarction (AMI), -blockers given within 6-
12 h can decrease the infarct size).
‒ They increase the diastolic (coronary) filling time.
‒ They cause redistribution of blood to the ischemic (subendocardial) regions.
‒ β-blockers improve myocardial metabolism through metabolic switching from
fat utilization to carbohydrates utilization (cytoprotective effect).

■ Cardiac arrhythmias (especially thyrotoxic and supraventricular arrhythmia).


‒ They decrease A-V conduction, lengthen the refractory periods of the SA node,
and suppress automaticity.
‒ Propranolol decreases excitability through its membrane stabilizing action.
‒ Acute arrhythmia during surgery is treated by i.v. esmolol.

■ Hypertrophic obstructive cardiomyopathy


‒ It is a congenital thickening of the ventricular wall and interventricular septum.
It is the most common cause of sudden death in young athletes. Thickening of
the interventricular septum impairs blood flow through the aortic outlet
especially during exercise.
‒ Drugs having –ve inotropic effect such as β-blockers and verapamil decrease
HR and contractility, thus decrease the outflow tract resistance. Drugs with +ve
inotropic effects (e.g. digoxin) have the opposite effect.

■ Hyperthyroidism: propranolol is used to control tachycardia, anxiety, and


tremors due to sympathetic overactivity in hyperthyroidism. It also prevents
peripheral conversion of T4 into T3.

■ Esophageal varices due to liver cirrhosis: propranolol is used to decrease


portal and hepatic blood flow through combined decrease of COP and inducing
VC in the splanchnic vascular bed (through an unopposed α-action).

■ Open angel glaucoma: topical timolol or betaxolol ↓ aq humor secretion.

■ Other uses: pheochromocytoma (must be combined with alpha-blockers); and


prophylaxis of migraine (propranolol has an unclear mechanism).

60
Adverse effects
Heart block
 Tiredness and fatigue (the most comm-
Heart block means block of the
on side effect) due to reduced COP and
electrical conduction at any point in
block of β2-mediated VD in skeletal the conducting system e.g. SA nodal
muscles (mainly non-selective agents). block, AV nodal block, or bundle
 Bradycardia and impairment of branch block.
myocardial contractility, so they can
precipitate heart failure or heart block
N.B.
in patients with compromised cardiac
Excessive myocardial depression
function. caused by overdose of β-blockers
 Bronchospasm in susceptible can be reversed by i.m. glucagon.
individuals due to blockade of β2- This is because β-blockers decrease
receptors which mediate dilation in the intracellular cAMP making all β-
agonists acting through cAMP is
bronchi. Asthma is an absolute
useless. Glucagon increases
contraindication for all beta-blockers.
contractility by a mechanism
 Aggravation of peripheral ischemia unrelated to cAMP.
and cold extremities (mainly non-
selective agents). (selective β-blockers
are the preferred class if there is associated peripheral vascular disease).
 In diabetic patients, β-blockers (mainly non-selective) can potentiate the
hypoglycemic effect of insulin and oral hypoglycemic drugs (because they block
glycogenolysis), and mask tachycardia & tremors resulting from severe
hypoglycemia.
 CNS effects: vivid dreams, night mares, and depression.
 Sudden withdrawal can increase the risk of angina and arrhythmias due to
adrenoceptor “supersensitivity”. Gradual withdrawal is recommended.

Contraindications

■ Absolute contraindications
 Bronchial asthma.
 Any degree of heart block.
 Prinzmetal’s (vasospastic) angina (see CVS).
 Sudden withdrawal after long-term use.

■ Relative contraindications
 Acute – or severe chronic heart failure.
 Peripheral vascular diseases (PVD).
 Diabetes mellitus.
 In athletes involved in strenuous sports because beta-blockers can interfere
with the ability to perform strenuous physical activities.

61
Part 4: Sympathoplegic drugs

█ CENTRALLY-ACTING SYMPATHOPLEGIC DRUGS

1. Alpha-methyldopa

Mechanism and pharmacological effects


In the CNS, α-methyldopa competes with dopa for dopa decarboxylase enzyme
leading to formation of α-methylnorepinephrine, (and also α-methyldopamine), which
is a false transmitter. α-methylnorepinephrine stimulates central α2 receptors
leading to reduced central sympathetic outflow and decreased BP.

Therapeutic uses
Methyldopa is the drug of choice to treat arterial hypertension in pregnancy
because of its long and reliable track record.

Adverse effects
 The most common side effect is sedation, nightmares, and mental depression
due to central deficiency of norepinephrine.
 Mild hyperprolactinemia and extrapyramidal manifestations due to central
deficiency of dopamine.
 Positive Coombs test and autoimmune hemolytic anemia.
 Autoimmune hepatitis is rare.

2. Clonidine

Mechanism and pharmacological effects


 Clonidine is a central α2-receptor agonist leading to decreased central
sympathetic outflow and reduction in the total peripheral resistance.
 Reduction of BP is not associated with changes of the RBF or GFR.

Therapeutic uses
 It is mainly used in the management of hypertension complicated by renal disease.
 To reduce anxiety accompanying opiate withdrawal or surgical operations.

Adverse effects
 Sedation and dry mouth (central effect).
 Sudden withdrawal of the drug can lead to rebound hypertension.
 Salt and water retention so it is usually combined with diuretics.

62
█ ADR
RENERGIC NEURON BLOCKERS
B S

Reserpine

Mecha
anism and pharmac
cological e
effects
 Reseerpine is a plant alkaloid thaat blocks vesicular
v
ake of th
upta he neurottransmitterrs norepinephrine,
dopaamine, an nd seroto onin in bboth cen ntral and
perip
pheral neurons, as well
w as adre enal medullla.
 Thesse transm mitters acccumulate in the neuronal
cyto
oplasm an nd are de egraded bby MAO enzyme,
leading finally to deple etion of thhe nervouss system
from
m these bio ogenic amines.
 The effect of reeserpine iss slow and
d persists for many
dayss after disc
continuatio
on.

Therap
peutic use
es
Trea
atment of mild-to
m mooderate hyypertension; howeve
er, it is now
w not con nsidered
amoong the firsst or second line drug
gs becausee of numerrous adverrse effects.

Advers
se effects
 The most impo ortant side
e effect is s
sedation, nightmare es, and meental deprression
(2%)) due to ceentral deple
etion of booth norepin
nephrine and serotonnin.
 Hype erprolactin
nemia and extrapyra amidal man nifestations
s (parkinso
onism) may occur
due to central depletion of dopamiine.
 GIT symptomss (abdomin nal crampss, mild diarrhea, increase HCl) are comm mon due
to ovverpredomminance of parasymp pathetic activity.

63
Part 5: Parasympathomimetic drugs (Cholinomimetics)

■ The parent compound of all cholinomimetic drugs is acetylcholine.


■ ACh is the natural neurotransmitter in the following sites:
‒ All autonomic ganglia whether sympathetic or parasympathetic.
‒ Parasympathetic nerve endings to involuntary organs and exocrine glands.
‒ Sympathetic nerve endings to thermoregulatory sweat glands.
‒ Sympathetic nerve endings to adrenal medulla.
‒ Skeletal muscle motor end plate.
‒ Certain tracts within the CNS
■ ACh acts on both muscarinic and nicotinic receptors to produce all the effects
listed in table 1, and table 3 (see before).
■ ACh is not used clinically because: (1) it has very short duration of action (seconds)
due to rapid hydrolysis by AChE enzyme; and, (2) it lacks selectivity.
■ Cholinomimetic drugs are drugs that produce effects similar to ACh or cholinergic
nerve stimulation, but with more selectivity and fewer side effects than ACh.

Classification of cholinomimetic drugs

Direct-acting cholinomimetics Indirect-acting cholinomimetics


They act by direct stimulation of They act by inhibition of AChE enzyme
cholinergic receptors leading to accumulation of ACh.

■ Muscarinic agonists ■ Reversible ChE inhibitors:


Bethanecol, carbachol Physostigmine, neostigmine,
Pilocarpine, cevimeline pyridostigmine, donepezil

■ Nicotinic agonists ■ Irreversible ChE inhibitors:


Nicotine, lobeline Organophosphate compounds

█ DIRECT-ACTING PARASYMPATHOMIMETICS

█ Muscarinic agonists:

Pharmacological effects:

CVS  ↓ AV conduction and HR (M2).


 VD of all vascular beds through release of EDRF (M3).
Respiratory  Contraction of bronchial smooth muscle (M3).
effects  ↑ bronchial secretions (M3).

64
Eye  Miosis due to contraction of constrictor pupillae muscle (M3).
 Accommodation for near vision due to contraction of ciliary
muscle (M3).
 ↓ IOP (contraction of the ciliary muscle causing opening of the
trabecular meshwork and facilitates drainage of aq humor).
GI tract  ↑ motility and relaxation of sphincters (M3).
 Salivation (M3) and increase HCl secretion (M1).
Urinary tract  Contraction of bladder smooth muscles (M3).
 Relaxation of sphincters (M3).
Exocrine gld  ↑ all exocrine secretions, salivation, lacrimation, sweating, etc.

1. Carbachol

 It is choline ester but resistant to hydrolysis by AChE enzyme.


 It stimulates both muscarinic and nicotinic receptors.
 It is used as local eye drops to ↓ IOP in glaucoma. It contracts the ciliary ms
causing opening of the trabecular meshwork and facilitates drainage of aq humor.

2. Bethanecol

 It is a choline ester but resistant to hydrolysis by AChE enzyme, so it has long


duration of action (2-3 h) as compared to Ach.
 It stimulates muscarinic receptors with no activity on nicotinic receptors.
 It is used to reverse post-operative urine retention and paralytic ileus (in absence
of organic obstruction).

N.B.
 Bethanechol is administered orally or s.c., not by i.v. or i.m., because parenteral
administration may cause cardiac arrest.
 Bethanechol is contraindicated to treat urine retention due to mechanical
obstruction of the bladder or intestine because increasing contraction against a
closed outlet can lead to rupture of the viscus.

3. Cevimeline and pilocarpine

 Cevimeline is synthetic drug – pilocarpine is a natural plant alkaloid.


 Both drugs act as muscarinic agonists with no nicotinic effects.
 Both drugs can be given orally to increase salivary secretion and decrease
symptoms of dry mouth (xerostomia) associated with Sjögren syndrome.
 Pilocarpine is used as local eye drops to ↓ IOP in glaucoma.

65
se effects of musca
Advers arinic agon
nists
‒ Mosst importaant side effects
e inc
clude nausea, vomiting, sweeating, salivation,
nchoconsttriction, hypotension
bron n, and diarrhea; all of
o which caan be bloc
cked by
atro
opine.

Contra
aindication
ns of muscarinic ag
gonists
‒ Pepttic ulcer
‒ Bron
nchial asthhma
‒ Hearrt block.

█ Nico
otinic ago
onists:

1. Nicotine

 It is a compon
nent of cig
garette sm
moke. It
is a poison with
w many adverse e effects
and no therap peutic benefit.
 The overall effects of o nicotin e are
commplex and d result from mixed
stimulation and inhibition o of all
autoonomic gan nglia:
‒ Sm mall dos ses stimu ulate autoonomic
gaanglia le eading to o hyperte ension,
ta
achycardia a, increase
e GIT perisstalsis,
in
ncrease HClH secrettion, and CNS
sttimulation.
‒ To oxic doses lead to hypotensio
h on and
C
CNS depre ession du ue to ga anglion
bllockade.
 Nicootine is adddictive substance. TTrans-
dermmal patch hes containing nicotiine are
usedd to help sm
mokers stopp smoking..

2. Varrenicline

 It is nicotinic receptor partial ag


gonist
used
d for smok king cessattion.
 Headache an nd nauseaa are the most
com
mmon adve erse effects
s.
 Conttraindicateed in pregnancy
p y and
brea
ast feeding.

66
█ INDIRECT-ACTING PARASYMPATHOMIMETICS
(Cholinesterase inhibitors)

Mechanism and pharmacological effects

 Indirect-acting parasympathomimetics inhibit AChE enzyme resulting in


accumulation of ACh and stimulation of both muscarinic and nicotinic receptors.
 They are classified, according to nature and duration of AChE inhibition, into
reversible and irreversible inhibitors.

█ Reversible AChE inhibitors:

They interact with AChE enzyme by making reversible bond allowing duration of
inhibition lasting from minutes to hours.

1. Physostigmine

 Natural plant alkaloid (tertiary amine) that is well-absorbed from the GIT and can
pass to CNS.
 It can reversibly inhibit AChE enzyme for 3-4 hours, leading to:
 Muscarinic effects: hypotension, bradycardia, salivation, lacrimation,
increased GIT peristalsis (diarrhea and colic), miosis, etc.
 Nicotinic effects: skeletal muscle contraction.
 Central effects: headache, insomnia, excitation, and convulsions.

Therapeutic uses
 Because of lack of selectivity and harmful CNS effects, it is usually used as local
eye drops to produce miosis and treat chronic glaucoma.
 Physostigmine can be used to reverse the central and peripheral manifestations
of atropine poisoning.

2. Neostigmine

 Synthetic drug (quaternary amine) that is poorly absorbed from the GIT and
cannot pass to CNS.
 It is similar to physostigmine in mechanism and effects but it has no CNS
actions.

Therapeutic uses
 To reverse postoperative urine retention and paralytic ileus. It is contraindicated
if there is mechanical obstruction (to avoid rupture of the bladder or intestine).

67
 To reverse postoperative muscle Myasthenia gravis
paralysis resulting from the use of non-
 Myasthenia gravis is an
depolarizing neuromuscular blockers.
autoimmune disease in which
 Treatment of myasthenia gravis: antibodies complex with
 Neostigmine not only increases ACh nicotinic receptors at the
level in the neuromuscular junction neuromuscular junction to
but also can directly stimulate nicot- cause skeletal muscle weakness
inic receptors at the motor end plate.  AChE inhibitors, such as
 Atropine could be given with pyridostigmine, are used to
neostigmine to block the unwanted increase ACh levels at the
muscarinic effects caused by neuromuscular junction to fully
activate the remaining
excessive ACh.
receptors.
 Myasthenia gravis can be
3. Pyridostigmine diagnosed using the Tensilon
test, which can also assess the
 Reversible AChE inhibitor similar to adequacy of treatment with
neostigmine. AChE inhibitors.
 It is more preferred than neostigmine in
the chronic treatment of myasthenia
Alzheimer’s disease
gravis because:
‒ It has more selective action on  Alzheimer’s disease is chronic
neuromuscular junction (fewer degenerative disease
characterized by progressive
unwanted muscarinic effects).
impairment of memory and
‒ It has longer duration of action than cognitive functions.
neostigmine.
 Pathologic changes include
increased deposits of amyloid
4. Edrophonium β peptide and abnormal
protein (tau protein) in the
 It acts as the same of neostigmine and cerebral cortex, leading to
pyridostigmine but has very short cerebral vascular lesions, and
progressive loss of
duration of action (5-15 minutes).
cholinergic neurons.
 It is used in the diagnosis of
myasthenia gravis and to differentiate  Although evidence for the
benefit of AChE inhibitors is
between muscle weakness due to
statistically significant, the
insufficient treatment of myasthenia, or clinical benefit from these
due to excessive treatment with AChE drugs is mild and temporary. 
inhibitors (Tensilon test).

 Tensilon test: small doses of edrophonium improve muscle strength in


untreated patients with myasthenia, but worsen muscle weakness if it was due to
excessive dose of AChE inhibitors (excessive ACh stimulation at the neuro-
muscular junction results in muscle weakness due to maintained depolarization).

68
5. Don
nepezil an
nd rivastigmine

 Theyy are AChE


E inhibitors
s that act m
more selec
ctively on central
c ChE enzym
AC me.
 Theyy are used
d to increas
se ACh levvels in the CNS and thus imprrove memoory and
cogn
nitive defic
cit associatted with A s disease (see box).
Alzheimer’s

█ Irrev
versible ChE
C inhib
bitors:
Organnophosphate comp pounds

 Theyy include:
 D
Drugs: echhothiophate eye dropps
 Insecticide
es: parathiion and maalathion
 N
Nerve gasses: sarin and
a soman n

 Orgaanophosph
hates are highly lip id soluble
e and rapidly absorb
bed by alll routes
inclu
uding the skin.
s Their CNS pene
etration is rapid and high.
 Theyy interact with AChE enzymee by makin
ng irreversible (covvalent) bo
ond (i.e.
phossphorylatio
on of the enzyme).
 As time passe es, the stre
ength of th
he bond inncreases, (a processs called “a
aging”),
and AChE bec comes irreversibly inhibited. (With mos st types o
of organopphosph-
ates,, 50% of th
he enzymee undego aaging after 3 hrs and 95% after 12 hrs).
 Once AChE is
i inhibited, ACh a
accumulate
es throughout the nervous system,
s
caussing musca
arinic and nicotinic ssymptoms.
 Echo
othiophatte is the only
o non-a
absorbable hosphate. It is available as
e organoph
miottic eye dro
ops for gla
aucoma. It s effect in the eye lasts for weeeks.

Manife
estations of
o organop
phosphate
e toxicity:
‒ CVSS: hypoten
nsion, bradycardia, sw
weating.
‒ Resspiratory: bronchosp
pasm, increease bronc
chial
secrretions, resspiratory ms
m paralys is.
‒ GITT: abdominal colic, diarrhea, an nd salivatioon.
‒ Eyee: severe miosis
m point pupil)), lacrimatiion.
(pinp
‒ CNS S: hallucinations, con nvulsions, and coma a.
‒ Skeeletal ms: twitches
t and
a fascicu ulation.
 Thee cause of death is re espiratory fa
ailure (block
ked
airw
way, paralyzzed respirattory ms & in
nhibited RC C).

Manag
gement
 Ensu
ure patent airway and d artificial respiration
n.
 Gasttric lavage
e and skin wash
w to re
emove the toxin.

69
 Intra
avenous no
ormal saline to raise BP.
 The triad: atro
opine – pralidoxime
e – diazepa
am

opine (2 mg
Atro g i.v. bolus
s)
- A
Atropine is non-selec
ctive musc
carinic blocker and can
c cross BBB to block
b all
m
muscarinic manifestaations of exxcess ACh centrally and perip pherally.
- C
Check pulsse and BP after 5 min n; if no res
sponse, rep peat the doose of atro
opine till
th
he HR is > 80 bpm and
a systolic c BP > 80 mmHg.
- T
The patien nt should be main ntained atropinized
a for 24-448 hrs because
b
o
organophossphates arre highly li pid solublee. So, it may
m dissolvve in body fat and
re
eleased ag
gain over tiime.

Pralidoxime (P
PAM; 2 gm
m i.v. over 2
20-30 min))
- Itt is also avvailable as ready-to-uuse autoinjjector.
- Iff given eaarly (before e aging), iit can rea
activate
(d
dephospho orylate) AC ChE enzy me espec cially at
th
he neurom muscular junction.
- P
Pralidoxime e is only efffective in o
organopho osphate
to
oxicity (i.e. it does not
n have a an effect if
i AChE
e
enzyme iss carbam mylated, a as occurs with
n
neostigmine or physo ostigmine).

zepam (10
Diaz ontrol convulsions.
0 mg i.v. orr i.m.): to co

5. Selective therapeu
Table 5 utic indicattions of pa homimeticss
arasympath

Clinica
al conditio
on D
Drug Recep
ptor
Postop
perative urine retentio
on B
Bethanechool (direct) M3
and pa
aralytic ileus N
Neostigmine (indirect)) M&N
Glauco
oma P
Pilocarpine, M3
C
Carbachol, physostigmine M&N
Xerosto
omia C
Cevemeline
e M3
Alzheim
mer’s disea
ase D
Donepezil, rivastigmin
ne M&N
Myasth
henia gravis N
Neostigmine, pyridosttigmine Nm
Diagno
osis of mya
asthenia E
Edrophoniu
um Nm
Atropin
ne toxicity P
Physostigm
mine M&N

70
Part 6: Muscarinic antagonists

Actions and chemical structure


They are either tertiary amine alkaloids or quaternary amines:
 Plant alkaloids: atropine is found in Atropa belladonna and scopolamine
(hyoscine) is found in Hyoscyamus niger. They are tertiary amines (i.e. well
absorbed and can pass to CNS).

 Synthetic derivatives: are either tertiary or quaternary amines (limited CNS


penetration):
 Drugs used mainly as bronchodilators: Ipratropium
 Drugs used mainly as antispasmodics: Hyoscine butylbromide
 Drugs used mainly to decrease HCl secretion: Pirenzepine
 Drugs used mainly for genitourinary system: Oxybutynin, tolterodine
 Drugs used mainly as mydriatics: Homatropine, tropicamide
 Drugs used mainly to treat parkinsonism: Benztropine

Mechanism and pharmacological effects


Muscarinic-receptor antagonists are competitive antagonists of ACh at all
muscarinic receptors.

CVS effects  They block M2 receptors in the SA node and increase HR.
 No significant effect on the force of contraction because there
are no muscarinic receptors, or parasympathetic innervation of the
ventricles.
 Blockade of vascular M3 receptors has no significant clinical
value.
 High doses cause toxic VD in the facial blush area (atropine flush)
which is not related to the antagonistic action.
Respiratory  Bronchodilatation and decrease mucus secretion.
GIT  Decrease salivation and HCl secretion.
 Decrease motility (antispasmodic action).
Urinary  Relaxation of the bladder smooth muscles and contraction of the
bladder sphincters leading to urine retention.
Sweat  Blocking of muscarinic receptors in thermoregulatory sweat glands
glands (cholinergic) leading to dry skin and elevation of body temperature
(atropine fever).
 Children are more sensitive to this effect.
Eye  Passive mydriasis due to paralysis of constrictor pupillae muscle.

71
 Cycloplegia (paralysis of ciliary muscle) leading to loss of
accommodation for near vision.
 Increase IOP due to mydriasis (decrease aqueous humor
drainage).
CNS  Tertiary amines can produce sedation, amnesia, delirium, and
hallucinations.

Therapeutic uses

CVS:
■ Bradycardia: parenteral atropine is the standard drug for most cases of
bradycardia including reflex bradycardia caused by vasopressor drugs.

Respiratory:
■ Bronchial asthma: Ipratropium is a quaternary amine. It has greater selectivity
for the bronchial tissue and limited CNS effects. It is given by inhalation to dilate
the bronchi and reduce secretions in asthma and chronic obstructive pulmonary
disease (COPD).
■ Preanesthetic medication: Preanesthetic injection of atropine is used in order
to:
 Prevent bronchoconstriction and reduce bronchial secretions caused by
excessive vagal stimulation during anesthesia.
 Protect the heart from excessive vagal tone (bradycardia) occurred during
anaesthesia.

GIT disorders:
■ Peptic ulcer: pirenzepine has greater selectivity for blocking M1 receptors in the
stomach and reduce HCl secretion; however, it is now rarely used because of
the availability of new and more potent drugs.
■ Diarrhea: the classic combination of atropine with diphenoxylate, (a congener
of meperidine), is available under many names (e.g, Lomotil). They decrease
hypermotility and secretions.
■ Abdominal colic: e.g. hyoscine butylbromide (Buscoban).

Urinary disorder:
■ Acute cystitis: oxybutynin is used to decrease bladder spasm and urinary
urgency associated with inflammatory bladder disorders.
■ Urine incontinence in adults: tolterodine is a new muscarinic antagonist used
for this indication because it has greater selectivity for bladder M3 receptors
and has long duration of action.

72
Eye:
■ Funduscopic examination: muscarinic antagonists are used as eye drops
(cyclopentolate; tropicamide) to produce mydriasis and cycloplegia and
facilitate retinal examination; however, phenylephrine (α-agonist) is preferred for
simple fundus examination due to its short duration.
■ Iridocyclitis: inflammation of the iris can cause adhesions between the iris and
lens (synechia). Long acting atropine eye drops is used to produce complete
cycloplegia and mydriasis (M3) to prevent this adhesion.

CNS:
■ Parkinson’s disease: benztropine has Motion sickness
greater selectivity for blocking the  It is a very common disturbance
muscarinic receptors in the basal of the inner ear that is caused by
ganglia and decrease the excitatory repeated motion such as from the
effect of ACh. movement of a car or ship.
 Bizarre head movement affects the
■ Motion sickness: scopolamine
organs of balance and equilibrium
(hyoscine) is the standard drug used for (vestibulocerebellar apparatus)
this indication. It blocks muscarinic causing nausea and vomiting.
receptors in the vestibulocerebellar  Overactivity of muscarinic
pathway that are responsible partially receptors is suspected to play an
for the nausea and vomiting. important role in this condition.

Other:
■ Organophosphate toxicity: atropine is the standard drug (see before).

Adverse effects
 Blurred vision (due to mydriasis and cycloplegia).
 Rise of IOP (glaucoma).
 Dryness of all body secretions: dry mouth, dry skin, dry eyes, etc..
 Urine retention especially in patients with senile enlarged prostate.
 Tachycardia.
 In children: atropine fever (due to blockade of thermoregulatory sweating
resulting in hyperthermia) and flush . Children are more sensitive to this effect.

Contraindications
 Narrow angle glaucoma
 Obstructive diseases of the GIT (e.g. pyloric stenosis), paralytic ileus, intestinal
atony of the elderly, etc.
 Urine retention due to senile enlarged prostate
 It should be used with caution in children.

73
Part 7: Ganglion blocking drugs

■ Trimethaphan and mecamylamine are competitive blockers of ACh at nicotinic


receptors at both sympathetic and parasympathetic ganglia.
■ Because of lack of selectivity and numerous adverse effects, they are used
rarely in the clinical setting hypertensive emergencies).

Part 8: Neuromuscular blockers

Non-depolarizing NMBs Depolarizing NMBs


members ■ Tubocurarine (prototype) is ■ Succinylcholine
rarely used clinically at this (It is ester of ACh)
time
■ Semisynthetic derivatives:
 Mivacurium
 Atracurium and
Cisatracurium
 Vecuronium
Absorption  All members are not absorbed orally because they are polar
and compounds (quaternary amines), they must be given parenterally.
distribution  All cannot cross BBB or placental barrier

Metabolism  Atracurium: spontaneous  By plasma pseudo ChE enzyme


plasma hydrolysis. Breakdown
products may cause seizures.
 Vecuronium: liver.
Duration  Mivacurium: 10-20 min  Few minutes.
 Atracurium: 20-30 min
 Vecuronium: 30 -40 min
Mechanism  Competitive block of Ach at  Depolarizing block of Nm
of action Nm receptors → muscle receptors → ms paralysis
paralysis. through 2 phases:
 Muscle paralysis can be  Phase 1: initial depolarization →
reversed by excess Ach transient ms contraction
(AChE inhibitors). followed by paralysis due to
maintained depolarization

74
(depolarizationn block).
 Phasse 2: the m muscle beccomes
repo
olarized agaain but remmains
insen
nsetive to sstimulation
n by
ds ↑ Ach to be
Ach (i.e. it need
stimu
ulated) (deesensitization
block
k).

Reverssal of  Neostigm
mine can re
everse the  Neosstigmine inncreases muscle
m
block block by in
ncreasingg Ach level paralysis during g phase 1 (due to
at NMJ annd displace
e increase muscl e depolarization),
competitiv
ve blockerss from the but itt can reverrse the bloock in
receptors. phas se 2 (becauuse the rec ceptor
is rela
atively inseensetive an
nd
needs excess A Ach to be
stimuulated).
 Freshh blood traansfusion.
peutic  To induce sk ms rela
Therap axation  The same
s but ssuccinylcholine is
uses during surrgical ope
erations. prefe
erred for shhort proced
dures
 To control convulsioons during e.g. endotrach
e heal intubaation
electrocon
nvulsive (E
ECT) (has shorter
s duaation).
therapy
Adversse  Histamine
e release lleading to  Sudd
den rise off IOP due to
t
effects
s hypotensio
on and contrraction of tthe extraoc
cular

75
bronchospasm. muscles in phase 1.
 Respiratory paralysis in high  Acute hyperkalemia (which
doses. may be dangerous and life-
threatening). It is due to efflux of
muscle K+ during depolarization.
 Postoperative muscle pain.
 Bradycardia due to stimulation
of cardiac muscarinic receptors
(similar to ACh).
 Prolonged respiratory
paralysis (apnea) may result
from congenital deficiency of
PsChE enzyme (treaed by
artificial respiraion and blood
ransfusion).
C/I and  Bronchial asthma: why?  Glaucoma or recent eye
precautions  Myasthenia gravis. surgery: why?
 With aminoglycosides or  Congenital deficiency of PsChE
quinidine (they can aggravate enzyme: why?
ms paralysis).

76
 
77
 
78
Review Questions

Mention the pharmacodynamic principles underlying the use of:

 Dopamine in shock.
 Adrenaline in acute anaphylactic shock.
 Ritodrine to delay premature labor.
 Sumatriptan in acute migraine.
 Ergometrine in postpartum hemorrhage.
 Tamsulosin in senile enlarged prostate.
 Propranolol (beta-blockers) in hypertension.
 Beta-blockers in obstructive cardiomyopathy.
 Alpha-methyldopa in hypertension of pregnancy.
 Neostigmine in myasthenia gravis.
 Pralidoxime (PAM) in organophosphate toxicity.
 Atropine before surgical operations.
 Tolterodine in urine incontinence in adults.
 Succinylcholine before endotracheal intubation.

Mention the pharmacodynamic principles underlying the contraindication of:

 Ergometrine (or ergotamine) during pregnancy.


 Alpha-blockers (or beta-blockers) alone in pheochromocytoma.
 Propranolol (beta-blockers) in bronchial asthma.
 Reserpine in parkinsonism.
 Bethanechol in urine retention due to senile enlarged prostate.
 Neostigmine to treat ms paralysis of organophosphate intoxicated patient.
 Atropine in closed angle glaucoma.
 Atropine in old patient with senile enlarged prostate.
 Atracurium in bronchial asthma.

Mention the rational of the following combinations:

 Alpha-blockers with beta-blockers in pheochromocytoma


 Atropine with neostigmine in myasthenia gravis.
 Caffeine with ergotamine in acute migraine.

Mention the main differences between:

 Dopamine and dobutamine.


 Ergotamine and ergometrine.
 Propranolol and atenolol.

79
Of each of the following questions, 5. Cholinergic stimulation causes:
select ONE BEST answer: A. Urine retention
B. Bronchodilatation
1. Regarding adrenergic α1 receptors, C. Sweating
all are true EXCEPT:
D. Tachycardia
A. Molecular techniques revealed the
presence of a number of subclasses.
E. Reduced gut motility
B. Their stimulation can contract the
pregnant human uterus.
6. Nicotinic acetylcholine receptors
are found in all the following sites
C. Their stimulation can increase EXCEPT:
peripheral resistance
A. Sympathetic ganglia
D. Their effect is more potent and shorter
duration than β2 receptors. B. Presynaptic nerve endings
E. Their activation leads to increase C. Central nervous system
intracellular calcium D. Skeletal muscles motor end plate
E. Vascular endothelium
2. Regarding adrenergic β2 receptors,
all are true EXCEPT: 7. Increased urinary levels of vanilyl
A. Their stimulation can relax the non- mandelic acid (VMA) above 8 mg/24
pregnant human uterus. hours is diagnostic marker of the
B. Their activation on mast cells leads to following tumors:
stabilization of mast cell membrane. A. Pheochromocytoma
C. Their activation leads to increase B. Carcinoid tumor
intracellular cAMP. C. Leukemia
D. Their selective antagonists have no D. Lymphoma
clinical uses. E. Astrocytoma
E. Continuous and prolonged stimulation
can lead to down-regulation 8. The actions of norepinephrine at
adrenergic receptors are terminated by
3. Stimulation of cardiac M2 which of the following:
cholinoceptors cause which of the A. Metabolism by MAO in the liver
following: B. Reuptake into the nerve terminal
A. Decrease myocardial contractility C. Conversion into 5-HIAA
B. Decrease SA nodal activity and heart D. Conversion to dopamine
rate E. None of the above
C. Decrease conduction velocity through
the Purkinje fibers 9. The following is true for true
D. Decrease coronary blood flow cholinesterase:
E. All of the above. A. Is found in autonomic ganglia and
myoneural junctions
4. Physiological events mediated by B. Is found in plasma and liver
stimulation of β1 adrenoceptors include C. It needs 2 weeks to be regenerated
all the following EXCEPT:
D. It can metabolize acetylcholine as well
A. Increase insulin secretion as other choline esters
B. Increase systolic blood pressure E. Its presence is not necessary for life
C. Shorten myocardial cell refractoriness
D. Increase outflow resistance in patients 10. Which of the following drugs acts
with obstructive cardiomyopathy indirectly by releasing norepinephrine?
E. Increase renin release by A. Angiotensin
juxtaglomerular cells of the kidney B. Dopamine

80
C. Phenylephrine E. Phenylepherine causes rise of blood
D. Amphetamine pressure with bradycardia
E. Isoprenaline
15. For the treatment of acute
11. Attention-deficit hyperactivity anaphylactic shock, adrenaline must be
disorder in children can be treated by: given by the following route:
A. Ephedrine A. Inhalation
B. Modafinil B. Subcutaneous
C. Tizanidine C. Intravenous
D. Methylphenidate D. Intramuscular
E. Midodrine E. Intracardiac

12. The following is correct about the 16. Regarding reflex bradycardia
action of sympathomimetics: induced by administration of
vasopressor drugs:
A. Adrenaline has almost exclusively β-
adrenoceptor agonist actions A. It starts to work as a compensatory
B. Noradrenaline has an approximately response after long time of
equal mix of α-and β-adrenoceptor vasopressor use
agonist actions B. Reflex is mediated through stretch
C. Isoprenaline has predominantly α- receptors in the left pulmonary artery
adrenoceptor agonist actions C. The receptors begin to respond at
D. Phenylepherine has predominantly β- pressure ≥ 150 mmHg
adrenoceptor agonist actions D. In chronic hypertension the set point
E. Dopamine acts on specific D- is shifted to a higher level
receptors as well as other E. Beta blockers readjust the set point to
adrenoceptors. a higher level

13. Epinephrine, all are true EXCEPT: 17. Dobutamine is best indicated for
A. It is a polar (ionized) compound. management of which the following
shock:
B. Is synthesized from norepinephrine
within the adrenal medulla A. Septic shock
C. Cannot be administered orally. B. Cardiogenic shock
D. It is available as eye drops for C. Anaphylactic shock
ophthalmic use. D. Hypovolemic shock
E. The final product of metabolism is E. Neurogenic shock
vanillylmandelic acid (VMA).
18. Ritodrine hydrochloride can be used
14. The following statements about the in the management of:
action of sympathomimetics (i.v.) are A. Parkinson’s disease
correct EXCEPT: B. Bronchial asthma
A. Adrenaline infusion causes rise in both C. Depression
systolic and diastolic blood pressure D. Premature labor
with tachycardia E. Bradycardia
B. Noradrenaline infusion causes rise in
both systolic and diastolic blood 19. Selective α2 agonists that is used to
pressure with bradycardia relieve muscle spasm associated with a
C. Dopamine infusion causes decrease variety of neurological conditions is:
in renal blood flow and GFR. A. Clonidine
D. Salbutamol causes fall of blood B. Tizanidine
pressure with tachycardia
C. Ritodrine

81
D. Midodrine 24. Which of the following drugs will
E. Alpha methyldopa decrease heart rate in a patient with a
normal heart but will have no effect on
20. Nasal decongestants carry the risk heart rate in a cardiac transplant
of cerebral stroke in which of the recipient?
following conditions: A. Epinephrine
A. Arterial hypertension B. Salbutamol
B. Allergic rhinitis C. Norepinephrine
C. Epistaxis D. Phenylephrine
D. Benign prostatic hypertrophy E. Dopamine
E. Sinusitis
25. False +ve test for antinuclear factor
21. Oxymetazoline has which of the may be caused by:
following actions: A. Phenoxybenzamine
A. Bronchodilation B. Prazosin
B. Vasoconstriction C. Reserpine
C. Hyperglycemia D. Yohimbine
D. Tachycardia E. Ergotamine
E. Inhibition of ejaculation
26. The following alpha blocker is best
22. Chronic orthostatic hypotension prescribed to decrease symptoms of
due to impaired autonomic reflexes can urine retention due to senile enlarged
be managed by: prostate:
A. Midodrine A. Prazosin
B. Ritodrine B. Tremazosin
C. Amphetamine C. Phenoxybenzamine
D. Modafenil D. Terazosin
E. Cocaine E. Tamsulosin

23. 65 year old male requires extensive 27. Alpha blockers can worsen which of
dental work. In your first session with the following urinary problems:
him you inject lidocaine (2%) plus l: A. Urine retention due to senile enlarged
100,000 epinephrine. Although there prostate
was initial anesthesia, you are B. Urine retention due to atonic bladder
surprised to discover that after 15 C. Urine retention with over flow due to
minutes the patient grimaces with pain spinal cord injuries
when you work in the affected area. D. Urine incontinence due to pelvic floor
What is the best possible explanation? pathology in women
A. The patient is a chronic complainer E. Dysuria and frequency associated
B. The injection missed the appropriate with bladder inflammation
nerves
C. The patient metabolizes lidocaine 28. All the following conditions can be
extra rapidly effectively treated by beta-blockers
D. The patient may suffer benign EXCEPT:
prostatic hypertrophy and is being A. Angina pectoris
treated with doxazosin B. Essential hypertension
E. In this patient lidocaine is an C. Raynaud’s disease
ineffective local anesthetic D. Open angle glaucoma
E. Supraventricular tachycardia

82
29. The therapeutic action of beta- 34. The following beta-blocker is
blockers in angina pectoris is believed preferred to control tachycardia when
to be primarily due to: peripheral vascular disease is also
A. Reduced production of associated:
catecholamines A. Propranolol
B. Dilatation of the coronary vessels B. Dilevalol
C. Decreased myocardial oxygen C. Timolol
requirement D. Pindolol
D. Increased peripheral resistance E. Sotalol
E. Increased sensitivity to
catecholamines 35. One of the following drugs is best
chosen for the control of hypertension
30. Beta-blockers are contraindicated during pregnancy:
in bronchial asthma because: A. Captopril
A. They produce bradycardia and fall in B. Propranolol
COP C. Reserpine
B. They increase bronchial secretions D. Phenoxybenzamine
C. They decrease pulmonary blood flow E. Alpha methyldopa
D. They increase airway resistance and
narrowing 36. Positive Coomb’s test and hemolytic
E. They inhibit the respiratory center and anemia may follow the administration of:
impair ventilation A. Prazosin
B. Alpha methyldopa
31. Myocardial depression caused by
overdose of beta blockers can be
C. Guanithidine
reversed by parenteral administration D. Reserpine
of: E. Clonidine
A. Adrenaline
B. Dopamine 37. One of the following drugs should
be avoided in the control of chronic
C. Isoprenaline hypertension associated with peptic
D. Glucagon ulcer:
E. Insulin A. Reserpine
B. Prazosin
32. Excessive bradycardia induced by
beta-blockers is best treated by:
C. Propranolol
A. Dopamine D. Clonidine
E. Alpha methyldopa
B. Epinephrine
C. Isoprenaline 38. The following statements about
D. Neostigmine pilocarpine are correct EXCEPT:
E. Atropine A. It is a natural plant alkaloid
B. It acts selectively on muscarinic
33. Essential tremors can be best receptors
decreased by which of the following beta
C. It can block the hypotensive effect of
blockers?
neostigmine
A. Atenolol D. It is not metabolized by AChE enzyme
B. Propranolol E. It has a clinically useful miotic action
C. Betaxolol
D. Nebivolol 39. The following statements about
E. Bisoprolol anti-ChE drugs are correct EXCEPT:

83
A. Physostigmine lowers IOP D. Postpartum urine retention
B. Neostigmine may be used with E. Acute cystitis
atropine to treat myasthenia gravis
C. Pyridostigmine have fewer visceral 45. Relatively selective muscarinic
side effects than neostigmine. blocker that is used to treat urine
D. Rivastigmine can be used to treat incontinence in adults is:
paralytic ileus A. Pyrenzepine
E. Edrophonium has short duration of B. Benztropine
action C. Ipratropium
D. Tolterodine
40. A central AChE inhibitor that is used E. Oxybutinin
to improve symptoms of Alzheimer’s
disease is:
46. The metabolites of which of the
A. Pyridostigmine following neuromuscular blockers can
B. Edrophonium lead to seizures?
C. Donepezil A. d-tubocurarine
D. Neostigmine B. Atracurium
E. Echothiophate C. Mivacurium
41. A short acting AChE inhibitor used D. Vecuronium
in the diagnosis of myasthenia gravis E. Succinylcholine
is:
A. Edrophonium 47. When succinylcholine is used to
B. Neostigmine provide muscle relaxation during
C. Pyridostigmine delivery by cesarean section, the
D. Rivastigmine following is true:
E. Donepezil A. It can cause fetal hypotonia and even
fetal paralysis
42. Which of the following drugs has B. It can relax the uterus and aggravate
the longest duration of AChE inhibition: postpartum hemorrhage
A. Echothiophate C. It can cause acute hyperkalemia and
B. Neostigmine arrest the heart of the fetus
C. Physostigmine D. It can cause maternal tachycardia
D. Pyridostigmine E. It can decrease the effect of general
E. Donepezil anesthetics

43. The cause of death in 48. The following statements are true
organophosphate toxicity is: for neuromuscular blockers EXCEPT:
A. Bradycardia A. Succinylcholine can cause
postoperative muscle pain.
B. Increased bronchial secretions
B. Atracurium undergoes spontaneous
C. Paralysis of the respiratory muscles plasma hydrolysis
D. Depression of the respiratory center C. Vecuronium breakdown products may
E. All of the above cause seizures.
D. Neostigmine can reverse muscle
44. All the following are known block caused by competitive blockers
contraindications for the use of
E. Synthetic derivatives are generally
atropine EXCEPT: preferred than d-tubocurarine
A. Closed angle glaucoma
B. Senile prostatic enlargement
C. Paralytic ileus

84
49. A muscarinic blockers that is used B. Pralidoxime is completely ineffective
as a standard treatment of motion for enzyme regeneration after aging of
sickness is: the enzyme.
A. Pirenzepine C. Pralidoxime is effective regardless
B. Oxybutinine AChE is phosphorylated (e.g. by
C. Atropine organophosphates) or carbamylated
(e.g. by neostigmine).
D. Scopolamine
E. Tolterodine D. Atropine should not be stopped
before systolic blood pressure rises
above 110 mmHg and pulse rate
50. Zolmitriptan produce above 100 bpm.
vasoconstriction of cerebral vessels
E. Diazepam should be given to reduce
and decrease pain mediators during
bronchospasm
acute migraine by acting on the
following receptor subtypes:
54. Regarding the management of a
A. 5HT 1B/1D patient with iridocyclitis, the following
B. 5HT 1E/1F is true:
C. 5HT 2A/2C A. Mydriatics are used to help drainage
D. 5HT 3 of exudative fluids from the anterior
E. 5HT 7 chamber of the eye.
B. Short acting mydriatics such as
51. Bethanechol, a direct acting phenylephrine are preferred to avoid
muscarinic agonist used for relieving prolonged blurring of vision
post-operative urine retention in C. Atropine is preferred because it
absence of organic obstruction, could produces complete cycloplegia and
not be given parenterally because: mydriasis
A. It can cause annoying salivation D. If the patient was a child below 12
B. It can cause cardiac arrest years old, atropine eye drops would
C. It can cause histamine release and be contraindicated.
severe anaphylaxis E. Physostigmine eye drops should be
D. It can cause urine leak out of control used to help drainage of aqueous
E. It can cause undesirable nausea and humor
vomiting
Answers
52. A muscarinic agonist given orally to
increase salivary secretion and
1D 12 E 23 D 34 B 45 D
decrease symptoms of dry mouth
2B 13 A 24 D 35 E 46 B
associated with Sjögren syndrome is:
3B 14 C 25 B 36 B 47 C
A. Cevimeline 4A 15 D 26 E 37 A 48 C
B. Carbachol 5C 16 D 27 D 38 C 49 D
C. Bethanechol 6E 17 B 28 C 39 D 50 A
D. Pyridostigmine 7A 18 D 29 C 40 C 51 B
E. Rivastigmine 8B 19 B 30 D 41 A 52 A
9A 20 A 31 D 42 A 53 B
53. Regarding the management of a 10 D 21 B 32 E 43 E 54 C
patient with organophosphate toxicity, 11 D 22 A 33 B 44 E
the following is true:
A. With most types of
organophosphates, 90% of the
enzyme undergoes aging within the
first 3 hrs.

85
 

86
 
 
 
 
 
 
 
 
 
 
 
 
 
 
 
 
 
 
 
 
 
Part 1
1: Basiic inform
mation

▌TUBU
ULAR FUN
NCTION AN
ND URINE
E FORMAT
TION

 Thee renal bloo


od flow (RB BF) is 1.1 L /min (~ 22%
2 of CO
OP).
 Thee glomerulaar filtration
n rate (GFRR) is 125 ml/min.
m
 Thee capillary tuft filtrate
es ~ 180 L of fluid per day. 99% of thhe filtered fluid is
reab
bsorbed ag gain during
g passage e in the renal tubules..
 Watter reabso
orption is usually
u 2ryy to Na+ re eabsorptioon (exceptt in the co
ollecting
ug that ↓ Na reabs
tubules; ‘CT’)). Any dru +
= ↑ Na lo
sorption (= +
oss), also ↓ water
bsorption (= ↑ water loss or ‘di uresis’).
reab

Tubular reabsorrption and sites of ac


ction of 3 ty
ypes of diuuretics

87
■ Proximal convoluted tubules (PCT):
 Reabsorption: (75% of the glomerular filtrate).
– Active reabsorption of Na+ (~65%).
– Passive (2ry to Na+) reabsorption of equiosmotic amount of water.
– Reabsorption of all filtered K+, glucose, amino acids, and drugs.
 Secretion: active secretion and reabsorption of organic acids and bases into
tubular fluid.

■ Loop of Henle (LOH):


 Descending limb: passive reabsorption of water due to hypertonicity of the
medullary interstitium.

 Ascending limb: active reabsorption of Na+ (~25%) (this causes hypertonicity


of the medullary interstitium), Ca2+ and Mg2+.

■ Distal convoluted tubules (DCT):


 Proximal part:
– Active reabsorption of Na+ (5-7%).
– Passive (2ry to Na+) reabsorption of equiosmotic amount of water.
– Active reabsorption of Ca2+ (under the influence of parathormone ‘PTH’).
 Distal part:
– Active reabsorption of Na+ (2–5%) in exchange with K+ (under the influence
of aldosterone).
– Passive (2ry to Na+) reabsorption of equiosmotic amount of water.

■ Collecting tubules (CT): Reabsorption of water under the influence of ADH.

▌EDEMA AND EDEMATOUS CONDITIONS

 Edema is defined as the accumulation of fluid in the interstitial space due to


either:
– Increased capillary hydrostatic pressure
– Decreased plasma oncotic pressure.
– Increased capillary permeability.
 Edema can be either exudative (having high protein content) or transudative
(having low protein content).
 Exudative edema results from increased capillary permeability as part of the
acute inflammatory response. It is usually localized to the site of inflammation
and will not be considered in this chapter.

88
 Transudative edema is usually generalized and is associated with renal Na+
retention. The three most common clinical causes are:
– Congestive heart failure (CHF): the decreased COP causes renal ischemia
which stimulates the renin-angiotensin-aldosterone system (RAAS) → Na+ and
water retention → edema.
– Liver cirrhosis: the cirrhotic liver cannot synthesize sufficient albumin and
other plasma proteins → ↓ plasma oncotic pressure. Hypoalbuminemia
together with portal hypertension and 2ry stimulation of RAAS cause fluid
retention (edema) and accumulation of fluid in the peritoneal cavity (ascites).
– Nephrotic syndrome: glomerular dysfunction causes excessive loss of plasma
proteins in urine → ↓ plasma oncotic pressure → edema.

Part 2: Diuretic classes and agents

Diuretics are drugs that increase urine volume and Na+ excretion.
Natriuretic: a drug that increase Na+ excretion by the kidney.

Classification of diuretics:

Renal diuretics E x t r a - r e n a l di u r e t i c s
They act directly on the kidney: They act indirectly on the kidney:
■ Thiazide diuretics: act on the ■ Water diuresis: ↑ water intake → ↓
proximal part of the DCT e.g. ADH release → diuresis.
hydrochlorothiazide. ■ Digitalis in CHF: ↑ the COP
■ Loop diuretics: act on the leading to ↑ RBF → diuresis.
ascending limb of loop of Henle e.g.
■ i.v. albumin in ascites or
furosemide.
nephrotic edema: to increase
■ K+ sparing diuretics: act on the plasma osmotic pressure →
distal part of the DCT e.g. mobilization of edema fluid toward
spironolactone. the vascular compartment → ↑ RBF
■ Osmotic diuretics: substances that → diuresis.
↑ the osmotic pressure of tubular
fluid → ↓ water reabsorption by renal
tubules e.g. mannitol.

N.B. Carbonic anhydrase inhibitors e.g acetazolamide: they are weak diuretics
that ↓ NaHCO3 reabsorption from the PCT and may cause metabolic acidosis. They
also ↓ aqueous humor secretion and can be used in the treatment of glaucoma (see
pharmacology of the eye).

89
█ Loop diuretics
(Furosemide, torsemide, bumetanide , and ethacrynic acid)

Pharmacokinetics
 They are absorbed from the GIT and secreted into the lumen of the PCT by an
organic acid excretory system.
 The absorption of furosemide is erratic but bumetanide is complete.
 Diuresis occurs within 5 minutes after i.v. administration and within 30 minutes of
oral administration.

Mechanism and pharmacological effects


 Loop diuretics inhibit Na+/K+/2Cl― co-transport system in the thick ascending
limb of LOH leading to inhibition of the active reabsorption Na+, Cl―, and K+.
These ions are excreted with equiosmotic amount of water.
– They also increase excretion of Ca2+, Mg2+, halides and H+.
– Na+ and water loss at this segment is high, so they are potent (or high
ceiling) diuretics (i.e., up to 25% of the filtered Na+ load).
 They ↑ renal PGE2 and PGI2 production leading to VD and ↑ RBF and GFR.
– Non-steroidal anti-inflammatory drugs (NSAIDs) inhibit PG synthesis and
antagonize this effect of loop diuretics.
– VD of pulmonary vascular bed also occurs due to ↑ PG formation.

Therapeutic uses
■ Edematous conditions: e.g. CHF, nephrotic syndrome, etc.
– Many patients require fluid and sodium restriction to have the best results.
– Diuretics are not used to treat edema due to lymphatic obstruction
(lymphedema) or inflammatory edema (localized edema with high protein
content is difficult to be resolved by diuretics).

■ Acute pulmonary edema: loop diuretic ↓ pulmonary congestion by:


– They cause venodilatation → ↓ venous return.
– They cause VD of pulmonary vascular bed even before diuresis occurs.
■ Acute renal failure: to maintain adequate GFR and enhance K+ excretion.
■ Acute hypercalcemia and acute hyperkalemia: saline should be given to
compensate for Na+ and water loss.
■ Hypertensive emergencies: i.v. furosemide is usually given in emergencies:
– Loop diuretics ↓ plasma volume.
– They cause peripheral VD due to ↑ PGs production in many vascular beds.
– Hyponatremia ↓↓ sensitivity of the vascular smooth muscles to circulating
catecholamines.

90
Adverse effects
– Hypovolemia and hypotension.
– Electrolyte disturbances: Hyponatremia, hypokalemia, hypomagnesemia, and
hypocalcemia (all need to be properly replaced).
– Hypokalemic metabolic alkalosis: due to ↑ tubular secretion of K+ and H+.
– Hyperuricemia and precipitation of acute gout: This is caused by:
– Increased uric acid reabsorption in the PCT as a result of hypovolemia (It may
be prevented by using lower doses to avoid hypovolemia).
– Competition with uric acid excretion at the organic acid excretory system in
the PCT.
– Ototoxicity:
– It is reversible hearing loss. It occurs with very high doses.
– It may be due impairment of ion transport in the stria vascularis (inner ear).
– Occurs more frequent with:
 Patients with impaired renal function.
 Ethacrynic acid.
 Concomitant use of other ototoxic drugs e.g. aminoglycosides.
– Allergic reactions: all loop diuretics (except ethacrynic acid) are derivatives of
sulfonamides; they cause occasional skin rash, eosinophilia, and less often,
interstitial nephritis.

█ Thiazide diuretics

Classification
 True thiazides (they are derivatives of sulfonamides): hydrochlorothiazide,
bendroflumethiazide.
 Thiazide-like diuretics: metalozone, indapamide, chlorthalidone.

Pharmacokinetics
 Thiazide diuretics are absorbed from the GIT. They are secreted into the lumen of
the PCT by an organic acid excretory system.
 They produce diuresis within 1–2 hours.

Mechanism and pharmacological effects


 Thiazides inhibit Na+/Cl― co-transport system in the proximal part of DCT
leading to inhibition of the active reabsorption Na+, Cl―. These ions are excreted
with equiosmotic amount of water.
– Excess Na+ reaching the DCT is reabsorbed in exchange with K+ (→ K+ loss).

91
– They also increase excretion of halides and H+.
– They ↓ Ca2+ excretion and enhance its reabsorption.
– Thiazides have moderate efficacy (i.e., maximum excretion of filtered Na+
load is only 5-7%).
– Most thiazides are ineffective if the GFR is < 30-40 ml/min (so it is not useful,
or even harmful, in presence of renal failure).

 The action of thiazides also depends on renal PGs like loop diuretics but to
much less extent.

Therapeutic uses
■ Mild edematous states: cardiac, hepatic, or renal (same as loop diuretics).
■ Essential hypertension (mild to moderate):
– They have the same mechanisms like loop diuretics (mention them).
– They are often combined with other antihypertensive drugs to enhance their
blood pressure-lowering effects.
■ Hypercalcuria and renal Ca2+ stones: to ↓ urinary Ca2+ excretion.
■ Nephrogenic diabetes inspipidus (DI):
– Thiazides can reduce urine volume in some cases of DI. This is called
“paradoxical antidiuretic action” and it is not clearly understood. It may be due
to improvement of ADH receptor sensitivity in the renal collecting tubules.

Adverse effects
– Hypovolemia and hypotension.
– Electrolyte disturbances: Hyponatremia and hypokalemia.
– Hypokalemic metabolic alkalosis: due to ↑ tubular secretion of K+ and H+.
– Hyperuricemia the same as with loop diuretics.
– Hyperglycemia: due to both ↓ pancreatic release of insulin and ↓ tissue
utilization of glucose.
– Hyperlipidemia: due to ↑ cholesterol and LDL (by 5-15%).
– Allergic reactions: thiazides are derivatives of sulfonamides; they cause
occasional skin rash, dermatitis, and less often, thrombocytopenia.

█ Potassium-sparing diuretics
(Spironolactone – triameterine – amiloride)

 Spironolactone is a steroid congener of aldosterone.


 Triamterene and amiloride are synthetic drugs but not steroids.

Pharmacokinetics

92
 All a
are absorb
bed from thhe GIT.
 Spironolactonne and triamterene a re metaboolized by th
he liver
 Amiloride is excreted
e unnchanged in the urine
e.
 Theey have slo
ow onset (days).
(

anism and pharmac


Mecha cological e
effects
 Site n: the disttal part of the DCT where
e of action w Na+ is
reabbsorbed (2–5%)
( in exchang e with K+ under the t
influ
uence of aldosterone.
 Spironolacto one is a compe etitive an ntagonist of
aldo
osterone at
a its recep
ptor site att the distall part of DCT
ding to ↑ Na
lead +
N excretio on (with exxcretion of equiosmo otic
+
amoount of waater) and K retention .
 Tria
amterene and amilo hibitors off Na+ channnels in the distal
oride are direct inh
partt of DCT leading to ↑ Na+ exccretion (witth excretio
on of equio
osmotic am
mount of
+
water) and K retention.

 The
e net effec
ct is:
– Mild
d Na+ and water loss s (i.e., max imum excrretion of filtered Na+ is only 2-5
5%)
– Hypperkalemiaa: due to ↓ K excretio
+ +
on (K will be retained in blood)).
– Mettabolic acidosis: duee to ↓ H io n excretion
+
n (H+ will be retained in blood).

Therap
peutic use
es
■ All c
cases of edema
e due to hyperraldostero
onism:
– Primary hyyperaldoste
eronism: e
e.g. Conn’s s disease.
– S
Secondaryy hyperaldoosteronism
m: e.g. in liv
ver cirrhos
sis or nephhrotic syndrome.

■ Use
ed in comb
bination with
w loop d
diuretics or
o thiazide
es in orderr to:
– T
To minimizze the risk of electrollyte imbala
ance:
Loop diuretics cause hypo okalemia while K+ sparing diuretics cause
hyperkalem
mia. Their combinati on can minimize elec
ctrolyte dissturbance..
– T
To minimizze the risk of acid-ba
ase imbala
ance:
Loop diuretics caus se metabo sis while K+ sparing
olic alkalos g diuretics
s cause
metabolic acidosis. Their
T comb
bination caan minimiz
ze acid-basse imbalan
nce.
– T
To make synergism
s in cases o f refractory
y (resistantt) edema.

■ Treatment off female pattern haiir loss:


Spirronolacton
ne is a weak compe etitive inhib ndrogens aat their receptors
bitor of an
d ↓ synthesis of testtosterone (antianderrogenic efffect). Som
and me dermatoologists
use
e this feature to stop androgen--related fro
ontal hair lo
oss in wom
men.

93
Advers
se effects
– Hyp a due to ↓ K+ excreti on.
perkalemia
– Hyp c metabolic acidos
perkalemic sis: due to ↓ K+
andd ↓ H+ excreetion.
– Spirronolactonee has antiandrog
a genic efffects
(gyn
necomastia a and impottence in ma
ales).

Contra
aindication
ns
■ All c mia: espec
cases of hyperkalem
h cially in the
e following
g conditionns:
– Patients with
w chronic c renal failu
ure.
– W
With drugss that caus
se hyperka alemia e.g. ACEIs.
■ Spironolacto one shou uld not be give en with carbenoxxolone because
b
carbbenoxolonne has ald
dosterone--like action
n and can
n antagonnize the effect of
spirronolactone.

Spironolac
S ctone Triamtere
ene - amiloride
Structu
ure Synthetic
S ssteroid Synthetic non-stero
oids
Metabo
olism Extensive
E m
metabolism
m in the Amiloride is excrete
ed
liiver unchangeed in urine
Mecha
anism of action
a Competitiv
C ve antagonism N +
Direct inh ibition of Na
with
w aldostterone at itts channels at the distal part
receptor sitte in the DCT
D of DCT
Antiand
derogenic efcts Gynecoma
G astia & impotence Not preseent

█ Osm
motic diuretics: Mannitol,, Glycerol

They arre chemica ubstances given by i.v.


ally inert su on in emerrgency con
i infusio nditions

Mecha
anism of action
a
 Firsst, they ↑ osmotic
o pressure off plasma leading to withdrawaal of transscellular
fluid
d (e.g. aqueous humo or, excesssive CSF, etc).
e
 Sec cond, they are freely filtered byy the glom merulus and ↑ osmottic pressure of the
tubu eading to ↓ water reab
ular fluid le bsorption by
b renal tub
bules.

Therap
peutic use
es
Acute congestiv ve glauco oma and a acute rise cranial prressure: th
e in intrac hey are
given b apid ↑ drain
by i.v. infussion for ra nage of off aqueous humor or CSF respectively
by increasing thee osmotic pressure
p o f the plasm
ma before diuresis beegins.

se effects:: dehydration with hyypernatrem


Advers mia is the main
m adverrse effect.

94
95
Advantages and disadvantages of diuretics in some
Part 3:
edematous conditions

█ CONGESTIVE HEART FAILURE (CHF)

Patients with CHF have ↓COP due to weak cardiac muscle, fluid retention, and lung
congestion. Many patients have also high blood pressure.

Advantages of diuretics:
– Correction of fluid retention.
– Lowering of blood pressure.
– Decrease preload (venodilatation and ↓ venous return) and afterload (due to
arterial VD) leads to improvement of cardiac contraction.
– Decrease lung congestion causes improvement of tissue oxygenation.
– Recent evidence showed that spironolactone reduces morbidity and
mortality rates in patients with advanced heart failure.

Disadvantages of diuretics: Recommendation:


– Excessive hypovolemia can ↓ COP. Combination
– Diuretic-induced acid-base and electrolyte between a K+
sparing diuretic and
imbalance may impair cardiac function.
loop diuretic is the
– Diuretic-induced hypokalemia can predispose to
best choice.
digitalis toxicity and cardiac arrhythmia.

█ CHRONIC KIDNEY DISEASES

The majority of patients with chronic renal diseases (e.g. chronic renal failure,
diabetic nephropathy, etc.) have fluid retention, hypertension, hyperkalemia, and
acidosis.

Advantages of diuretics:
Recommendation:
– Correction of fluid retention. Loop diuretics are
– Reduction of hyperkalemia. best choice.
– Reduction of hypertension.

Disadvantages of diuretics:
– Thiazides are ineffective when GFR is <30 ml/min, moreover it may be harmful.
– K+ sparing diuretics are contraindicated because they can exacerbate
hyperkalemia and acidosis.
– Carbonic anhydrase inhibitors (acetazolamide) are contraindicated because
they can exacerbate acidosis.

96
█ LIVER
R CIRRHOSIS

Patientts with chronic


c liv
ver diseasse have fluid
retentio
on, ascite
es, hyperrammonem mia and 2ry
hyperaldosteronissm.

Adva
antages of
o diuretics s:
– C
Correction of fluid rettention.
– S
Spironolacttone antag gonizes ald
dosterone.

Disa
advantagees of diure
etics:
– AAggressive
e use of diuretics
d ccan precipitate
h
hepatorena
al syndrom
me.
– AAggressive
e use of diuretics
d ccan precipitate
h
hyperammo onemia and heppatic
e
encephaloopathy. Hoow?

– N
Normally, the urine pH is ac
cidic (~5.6
6). In
a
acidic uriine, most of the e absorb bable
ammonia (NH3) is converted
a c into the non-
a
absorbablee ammoniu um ions (N
NH4+) and so it
iss removed
d out by th
he kidney (this is kn
nown
a
as ammoniia trapping
g).

– D
Diuretics cause
c hypokalemia and metabolic
a
alkalosis. This
T leads to:
t

 Systemic alkalosiis causes the pH off the


urine to becom me less acidic. This
decreasses conveersion of the urina ary absorb bable NH33 into the non-
absorbaable NH4. The excreeted NH3 is s thus reab
bsorbed ag gain from urine to
blood → hyperammmonemia a.
 Systemic alkalosis
s increasess the entry
y of NH3 in
nto the braain cells.

█ LOW
WER LIMB EDEMA
E DURING PRE GNANCY

– L
Lower limb
b edema during late pregnancy mon conditiion and is usually
y is comm
b
benign (physiologic). It occurs due to ho mbalance, and comp
ormonal im pression
o
of pelvic ve
eins by the
e enlarged uterus.
– U
Unilateral leg edema, redness, warmth, and
a tenderness requuire evalua ation for
d
deep venou us thrombosis (DVT) .
– P
Physiologicc edema canc educed by elevating the lowerr extremities and
be re
w
wear elastic stockinggs.

97
– Diuretics are better avoided during pregnancy because they effectively
reduce maternal plasma volume and consequently may reduce amniotic
fluid and/or placental blood flow.

Part 4: Volume depletion and fluid replacement

 Volume depletion can be caused by loss of blood or other body fluids e.g.
vomiting, diarrhea, etc.
 In cases of mild volume depletion, resuscitation can be adequately achieved with
oral fluid alone. Sodium chloride tablets and electrolyte-containing solutions are
often used.
 In cases of severe dehydration, i.v. fluid therapy is preferred and may be life-saving.
 Water alone is not an appropriate fluid for volume resuscitation since it enters the
cells by osmotic effect. Only one third of each administered liter remains in the
extracellular space, and only one twelfth of each administered liter remains in the
intravascular space.
 When electrolyte disturbances are present, the fluid used for resuscitation should
be chosen to correct both volume depletion and electrolyte disturbances.

█ Crystalloid solutions

■ Sodium chloride solutions:


 Normal saline (0.9% NaCl): It is the most commonly used solution. It contains
154 mEq sodium per litre, a concentration similar to the sodium concentration of
plasma. Due to the relatively high chloride content, normal saline carries a risk of
inducing hyperchloraemic metabolic acidosis when given in large amounts.

 Hypotonic saline (0.45% NaCl): contains 77 mEq sodium per liter, and can be
used when there is dehydration with hypernatraemia. In these patients, 5%
dextrose in water can be given simultaneously with normal saline.
 Hypertonic saline (3% NaCl): contains 513 mEq sodium per liter, and can be
used for management of acute hyponatremia.

■ Lactated Ringer’s solution:


 It is an isotonic solution containing sodium, potassium, chloride, calcium and
lactate. The lactate is metabolized by the liver into bicarbonate, which can help
correct metabolic acidosis. In lactic acidosis and liver disease this conversion is
impaired, so lactate-containing fluids should be avoided.
 It is not suitable for maintenance therapy because the Na+ and K+ contents are
too low to compensate for daily electrolyte requirement.

98
■ Glu
ucose (Dex
xtrose) solutions:
 Variious conceentrations are availab
ble e.g. 5%
%, 10% and 25%. Thhe 5% dex xtrose in
watter (also kn
nown as D55W) is isottonic and is
s the mostt commonlly used.
 Hyp
pertonic glucose solutions (ab bove 5%) should be b infused very slow wly and
cau
utiously to avoid
a hype
erosmolarr syndrom me and life--threateninng dehydra
ation.

█ Collo
oid solutions

 Colloids are classified


c as either n
natural (albumin and fresh frrozen plassma) or
artifficial (starc
ch and dexxtran).
 Theey preserve a high colloid
c osm
motic pre essure in the
t blood and theoretically
dessigned to increase
i thhe intravasscular volu
ume with much lesss effect on
n tissue
watter. Howe ever, collo
oid solut ions are a less-p preferred choice for
f the
man nagement of volume e depletionn because they are very expenssive and have not
shoown a morttality beneffit over iso
otonic salin
ne.

█ Orall rehydrattion thera


apy (ORT
T)

 Oraal electrolyyte solution


ns are useed in childdren, particularly witth gastroe
enteritis.
Thiss product contains sodium,
s po
otassium, chloride,
c citrate,
c and
d dextrose e, and is
dessigned to replace th he electrolyytes and water thatt are lost with vom miting or
diarrrhea.
 Glucose is typ pically add
ded to thesse oral repllacement solutions
s too promotee uptake
of ssodium via the intestiinal sodium
m/glucose co-transporter mechhanism.

Part 5: Dis
sorders of serum
m sodiu
um and potassiu
p um

█ Hypo
onatremiia and SIA
ADH

 Hypponatremia a is defineed as seru um Na+ <135 mEq/L L. It can bbe caused by any
med dical illnesses, such as CHF, livver failure, renal failu
ure, pneummonia, or SIADH.
 Sevvere hypon natremia (N +
Na <120 mEq/L) lea ads to fall of plasmaa osmolality, with
movvement off water fro om plasma a to brain and otherr cells cauusing neuro ological
man nifestations (altered mental sta atus, weakkness, neu uromusculaar irritabilitty, focal
neuurologic de eficits, com
ma or seizu res).
 ADH H (or vaso opressin) is released d from poosterior pittuitary in rresponse to high
plassma osmolality. It bin nds to threee receptors: V1a in the
t vascul ature (VC), V1b in
the brain, and d V2 in renaal collectin ↑ water abs
ng ducts (↑ sorption).
 Whe
en the AD
DH system is working y, ‘the urine should rreflect the blood’,
g properly
i.e. concentra
ated urine occurs
o whe
en plasma
a osmolality
y is high, aand vice ve
ersa.

99
 Man
ny factorrs (includding drug gs and other o non-
pha
armacologiical condittions) can
n stimulate e release of
ADH
H irrespecctive of plasma
p ossmolality, leading to
t
hyp a, a cond
ponatremia wn as “sy
dition know yndrome of
o

inap
ppropriate cretion” orr SIADH.
e ADH sec

Manag
gement of SIADH

 Fluiid restriction (1L/d) is the main


n line.
 Dem ay be “rarely” used; they
meclocyclline and lithium may t impai r the response of
the collecting ducts to ADH
A (by a non-recep
ptor mecha
anism).

 Vas sts: “Vapta


sopressin receptor antagonis ans”:

 AAntagonism of the V2 V recepto rs results in aquares sis, a uniquue electroly


yte-free
wwater excrretion by th he kidneyss.
 CConivapta an is a mixxed V1a an nd V2 antaagonist, an
nd tolvapta an, a selec
ctive V2
aantagonistt.
 TTolvaptan n is appro oved for treating hyponatrem
h mia assocciated with h heart
ffailure, cirrrhosis, and
d SIADH.
 Recent sttudies sho owed thatt vaptans may be more effeective in treating
hypervolem mia in hearrt failure th
han diuretic
cs.

█ Hype
ernatrem
mia

 Hyppernatremia is defineed as a pla sma Na+ >145


> mEq//L, and rep
presents a state of
hyp
perosmolality
 Hyppernatremia may be caused byy a primary y Na+ gainn or a watter loss, th
he latter
ng much more com
bein mmon. Re enal wate er loss results from
m either osmotic
o
diurresis or diiabetes ins
sipidus (D
DI).
 Hyppernatremia results in contracttion of brain cells
as water sh hifts to attenuate
a the risingg ECF
osm
molality. Th hus, the most
m seveere sympto oms of
hyp
pernatremia a are neuro
ological maanifestatio
ons.

Manag
gement of symptom
matic hype
ernatremia
a

 The
e mainstay of manage ement is th
he adminis
stration
of water, prreferably by b mouth h or nasoogastric
tube
e. Alternattively, 5% dextrose e in water (D5W)
can
n be given intravenou usly.
 Speecific thera
apy of the underlying
u cause.

100
█ Hypo
okalemia
a

 Potassium is the major intracellula ar cation. 98% of K+ in the bo dy is found in the


intra
acellular co
ompartment, leaving g 2% in exttracellular fluid spacees.
 Ren +
nal K excrretion occu urs from thhe DCT and d is mediated by ald a Na+
dosterone and
delivery to thee distal nep
phron.
 Hyppokalemia is defined as serum K+ <3.5 mEq/L.m It ca
an result fro
rom diminisshed K+
ake, transcellular shift of K+, or increased K+ loss.
inta
 Thee most com mmon ma anifestatio ons are mu uscle weakness, craamps, flatttened T
wavve and pro olonged QT T interval in
n the ECG..
 Bec +
cause K iss usually ex xchanged with H+ att the DCT, hypokalem mia is often
n linked
to m
metabolic alkalosis.
a Anyone
A of these two conditions s can lead to the othher.

Druugs cause transcellu of K+ (from


ular shift o m plasma to
t tissue)::
Insu eta-2 agonists: they ↑ transmem
ulin and be a+/K+-ATPaase activity
mbrane Na y.

ugs cause renal loss


Dru s of K+:
Min
neralocortic
coids, gluc
cocorticoid
ds, diuretic
cs, amphottericin-B.

Dru o K+: laxa


ugs cause GIT loss of atives

Manag
gement

 Patients with a K+ level of


o 2.5-3.5 mEq/L ma nly oral K+ rreplaceme
ay need on ent
 If th +
he K level < 2.5 mE +
Eq/L, intravvenous K should be e given. Thhe rate of infusion
i
shoould not ex
xceed 20 mEq/hr,
m Raapid IV infu
usion may cause
c seri ous arrhythmia or
eveen cardiac arrest.
a
 Hyp pomagnese emia is frequently
f associate ed with hypokalem
h mia. Concomitant
mag gnesium deficiency
d aggravattes hypok kalemia and rendeers it diffiicult to
+
treaatment by K alone.

█ Hype
erkalemia

 Hypperkalemia a is defined
d as serumm K+ >5 mEq/L.
m It can
c result ffrom trans scellular
+ +
shifft of K , or decreased d renal exc
cretion of K (as in chhronic renaal failure).
 Thee most co ommon manifestati
m ions are muscle paralysis, p palpitations, high
peaaked T wavve and sho ort QT interrval in the ECG.
 Beccause K is +
i usually exchange ed with H+ at the DCT,D hypeerkalemia is i often
linkeed to meta abolic acid
dosis.

Druugs cause transcellu of K+ (from


ular shift o m tissue to
o plasma)::
Insu ency and β-blockers: they ↓ transmembra
ulin deficie ane Na+/K++-ATPase activity.
a

ugs that ↓ renal excrretion of K +:


Dru
K+ sparing diu
uretics, AC
CEIs, NSAIIDs, cyclos
sporins.

101
Management

 Mild hyperkalemia: could be corrected by diuretics and oral cation exchange


resins (Polystyrene sulfonate) to promote the exchange of Na+ for K+ in the GIT.

 Severe hyperkalemia with ECG changes:


 Intravenous calcium gluconate to reduce cardiac toxicity (↓ membrane
excitability). The usual dose is 10 mL of a 10% solution infused over 2 to 3
minutes.
 Intravenous insulin with glucose: 20 U regular insulin mixed with 500 ml D5W.
 Correct metabolic acidosis with i.v. NaHCO3 solution.
 Hemodialysis is reserved for patients with renal failure or with life-threatening
hyperkalemia resistant to other treatment.

Part 6: Pharmacological manipulation of the urine pH

Normal urine pH is 5.2-6.5. It is possible, by the use of pharmacological agents, to


produce urinary pH values ranging from ~ 5 to 8.5.

█ Alkalinization of the urine

 Indications:
 To enhance excretion of acidic drugs and organic compounds e.g. aspirin,
sulfonamides, and uric acid.
 To enhance dissolution of uric acid and cystine stones.
 To relieve dysuria (burning micturition) in some cases of bladder infection.

 Alkalinizing agents:
 Oral: sodium and potassium citrate salts: citrate is metabolized into
bicarbonate which is excreted in urine.
 Intravenous bicarbonate solution: contains 5% NaHCO3.

█ Acidification of the urine

 Indications:
 It is rarely used clinically except in a specialized test to discriminate between
different kinds of renal tubular acidosis.
 It can be very dangerous in cases of renal or hepatic impairment.

 Acidifying agents:
 Oral: ascorbic acid > 2 g/d.
 Intravenous ammonium chloride (NH4Cl) solution.

102
 
103
 
104
Review Questions

Mention the pharmacodynamic principles underlying the use of:

 Furosemide in arterial hypertension.


 Furosemide in acute pulmonary edema.
 Thiazides in diabetes insipidus.

Mention the pharmacodynamic principles underlying the contraindication of:

 Furosemide in acute hyperuricemia.


 Thiazides in uncontrolled diabetes mellitus.
 Spironolactone in chronic renal failure.
 Amiloride with ACEIs.
 Ethacrynic acid with aminoglycosides.
 Furosemide with NSAIDs.

Mention the advantages and disadvantages of diuretics in the following


conditions:

 Congestive heart failure.


 Chronic kidney disease.
 Chronic liver disease.

Mention the rational of the following combination:

 Furosemide with spironolactone.

Mention 3 differences between:

 Furosemide and spironolactone.

105
Of each of the following questions, 6. Which of the following diuretics
select THE ONE BEST answer: can enhance the parathormone-
mediated calcium reabsorption from
1. The ascending part of the loop of the distal renal tubules:
Henle is the principal site of action of A. Hydrochlorothiazide
the following diuretics: B. Triamterine
A. Hydrochlorothiazide C. Amiloride
B. Triamterine D. Bumetanide
C. Amiloride E. Spironolactone
D. Bumetanide
E. Spironolactone 7. Idiopathic calcium urolithiasis
(hypercalciuria) can be treated by:
2. Hyperkalemia is a contraindication A. Hydrochlorothiazide
of the following diuretics: B. Ethacrynic acid
A. Furosemide C. Furosemide
B. Bumetanide D. Triamterine
C. Ethacrynic acid E. Bumetanide
D. Chlorothiazide
E. Spironolactone 8. Spironolactone is characterized by:
A. It interferes with aldosterone synthesis
3. Loop diuretics are clinically useful B. It competitively inhibit aldosterone
in the treatment of all the following action in the distal part of the distal
edematous states EXCEPT: renal tubules
A. Edema caused by congestive heart C. It inhibits sodium reabsorption in the
failure proximal renal tubules
B. Edema caused by chronic liver failure D. It is more potent diuretic than
C. Lymphedema hydrochlorothiazide
D. Nephrotic syndrome E. It has rapid onset and short duration
E. Ankle edema due to chronic
hydralazine treatment 9. Hydrochlorothiazide is clinically
useful in the treatment of all the
4. Intravenous albumin is the ideal following conditions EXCEPT:
choice for treatment of the following A. Edema caused by congestive heart
conditions: failure
A. Ascites due to chronic liver disease B. Edema caused by chronic liver failure
B. Edema due to chronic kidney disease C. Edema caused by chronic renal failure
C. Edema due to congestive heart failure D. Hypertension with or without edema
D. Lymphedema E. Recurrent calcium urolithiasis
E. Inflammatory edema
10. Adverse reactions associated with
5. Vigorous diuretics are thiazide therapy include all the
contraindicated in resistant ascites due following EXCEPT:
advanced liver disease because: A. Hyperglycemia
A. It can lower blood pressure to a B. Hyperuricemia
critical level C. Metabolic acidosis
B. It can precipitate hepatorenal D. Fluid and electrolyte imbalance
syndrome E. Hypotension
C. It can lead to severe dehydration
D. It can decrease ascetic fluid suddenly 11. All the following diuretics can
and drastically aggravate digitalis toxicity EXCEPT:
E. It aggravate hypoalbuminemia A. Hydrochlorothiazide

106
B. Furosemide B. Hyperuricemia
C. Bumetanide C. Metabolic alkalosis
D. Amiloride D. Fluid and electrolyte imbalance
E. Ethacrynic acid E. Hypercalcemia

12. Adverse effects of loop diuretics 17. Acute pulmonary edema is best
include all the following EXCEPT: treated by i.v. administration of:
A. Magnesium deficiency A. Hydrochlorthiazide
B. Sodium deficiency B. Furosemide
C. Hypoglycemia C. Mannitol
D. Hypovolemia D. Amiloride
E. Hyperuricemia E. Metalozone

13. Hypokalemia can be caused by all 18. Which of the following diuretics has
the following drugs EXCEPT: the highest potential to cause
A. Captopril ototoxicity:
B. Salbutamol A. Chlorothiazide
C. Thiazides B. Furosemide
D. Corticosteroids C. Ethacrynic acid
E. Insulin D. Acetazolamide
E. Spironolactone
14. The following statements are true
concerning the precautions during the 19. All the following are uses of loop
use of diuretics in different metabolic diuretics EXCEPT:
disorders EXCEPT: A. Acute pulmonary edema
A. Furosemide may enhance digitalis B. Severe hypertension
toxicity in congestive heart failure C. Acute hypercalcemia
B. Furosemide may aggravate D. Acute oliguria
hyperammonemia in chronic liver E. Calcium urolithiasis
failure
C. Chlorothiazides may aggravate renal 20. In an addisonian patient, all of the
impairment in chronic renal failure following agents would have diuretic
D. Thiazides may aggravate action EXCEPT:
hyperglycemia in diabetes mellitus A. Mannitol
E. Thiazides may increase formation of B. Chlorothiazide.
urinary uric acid crystals in chronic C. Bumetanide
gout
D. Furosemide.
E. Spironolactone
15. All the following drugs can produce
salt and water retention after their
prolonged use EXCEPT:
21. Gynecomastia may occur with the
use of the following diuretics:
A. Nifedipine
A. Chlorothiazide
B. Minoxidil
B. Furosemide
C. Amiloride
C. Amiloride
D. Prazosin
D. Acetazolamide
E. Hydralazine
E. Spironolactone
16. Adverse reactions associated with
furosemide therapy include all the
22. The most dangerous complication
of injudicious use of diuretics in
following EXCEPT:
patients with advanced liver diseases
A. Hearing loss is:

107
A. Aggravation of hypotension and 27. A 64-year-old woman with
fatigue congestive heart failure. She complains
B. Electrolyte imbalance of swelling in her legs and ankles. The
C. Acid-base imbalance doctor decides to increase her level of
D. Precipitation of hepatorenal syndrome diuretics. What complication should the
E. Marked dehydration doctor be most aware of for this
patient?
23. Acute congestive glaucoma is best A. Diuretic-induced metabolic acidosis
treated by i.v. administration of: B. Hepatic encephalopathy
A. Bumetanide C. Hypercalcemia
B. Furosemide D. Hyperkalemia
C. Mannitol E. Hypokalemia
D. Amiloride
E. Metalozone 28. One of your clinic patients is being
treated with spironolactone. Which of
24. The following statements the following statements best
concerning hypokalemia are true describes a property of this drug?
EXCEPT: A. Contraindicated in heart failure,
A. It is a side effect predicted with all especially if severe
diuretics B. Inhibits Na+ reabsorption in the
B. It is commonly seen in patients with proximal renal tubule of the nephron
hyperaldosteronism C. Interferes with aldosterone synthesis
C. It can be manifested by ECG changes D. Is a rational choice for a patient with
D. It could be prevented by the use of K+ an adrenal cortical tumor
sparing diuretics E. Is more efficacious than
E. It is a risk factor for digitalis toxicity hydrochlorothiazide in all patients who
receive the drug
25. The best intravenous agent given
to patients with advanced liver disease 29. A patient taking an oral diuretic for
to correct ascites and edema is: about 6 months presents with elevated
A. Human albumin fasting and postprandial blood glucose
B. Mannitol levels. You suspect the glycemic
C. Furosemide problems are diuretic-induced. Which
of the following was the most likely
D. Chlorothiazide
cause?
E. Spironolactone
A. Acetazolamide
B. Amiloride
26. A 63-year-old man presents to the
emergency department with worsening
C. Chlorothiazide
heart failure. Physical exam reveals D. Spironolactone
pitting edema in his ankles. Past E. Triamterene
medical history is significant for an
allergic reaction following exposure to 30. Chlorthalidone and torsemide are
trimethoprim–sulfamethoxazole. Which members of different diuretic classes,
drug should the physician prescribe to in terms of mechanisms of action, but
him? they share the ability to cause
A. Acetazolamide hypokalemia. Which of the following
B. Ethacrynic acid statements best describes the general
C. Hydrochlorothiazide mechanism by which these drugs
cause their effects that lead to net
D. Mannitol
renal loss of potassium?
E. furosemide
A. Act as aldosterone receptor agonists,
thereby favoring K+ loss

108
B. Block proximal tubular ATP- C. Lupus
dependent secretory pumps for K+ D. Ototoxicity
C. Increase delivery of Na+ to principal E. Hyperuricemia
cells in the distal nephron, where
tubular Na+ is transported into the
cells via a sodium channel in
exchange for K+, which gets Answers
eliminated in the urine
D. Inhibit a proximal tubular Na,K- 1D 11 D 21 E 31 B
ATPase such that K+ is actively 2E 12 C 22 D 32 C
pumped into the urine 3C 13 A 23 C 33 B
E. Lower distal tubular urine osmolality, 4A 14 E 24 A 34 A
thereby favoring passive diffusion of 5B 15 C 25 A
K+ into the urine 6A 16 E 26 B
7A 17 B 27 E
31. Which of the following is a clinical 8B 18 C 28 D
indication for use of Mannitol? 9C 19 E 29 C
A. Chronic simple glaucoma 10 C 20 E 30 C
B. Cerebral edema
C. Pulmonary edema
D. Acute heart failure
E. chronic renal failure

32. All the diuretics act from the


luminal side of the renal tubule
EXCEPT:
A. Torsemide
B. Hydrochlorothiazide
C. Spironolactone
D. Chlorthalidone
E. Furosemide

33. A 55-year-old male with kidney


stones has been placed on a diuretic to
decrease calcium excretion. However,
after a few weeks, he develops an
attack of gout. Which diuretic was he
taking?
A. Furosemide.
B. Hydrochlorothiazide.
C. Spironolactone.
D. Triamterene
E. Acetazolamide

34. Your 60 year old male hypertensive


patient who had a myocardial infarction
a year ago is now showing signs of
CHF. You therefore add spironolactone
to his drug regimen. What side effect
should you warn him about?
A. Gynecomastia
B. Hypokalemia

109

You might also like